Thread Rating:
  • 0 Vote(s) - 0 Average
  • 1
  • 2
  • 3
  • 4
  • 5
NBME free download - serena
#1
hi
does anyone hav nbme free version posted heer on this forum,i saw it 20 days back, but am unable to find it now.if somwon has it can u pls post link here,
thanks
Reply
#2
anyone
Reply
#3
it is there, but i cant find it despite efforts.does anyone hav it, triple helix, cricoid, hellerphd,, anyone
Reply
#4
Block 1
================================================================================
1. An 83-year-old woman who has dementia, Alzheimer type, is brought to the office for a return visit by her daughter, with whom she lives. While intermittently somewhat confused, until recently the patient had been able to handle most of her activities of daily living. In the past month, however, she has shown little interest in eating and is awake most of the night. The daughter says the patient has been seeing things, especially at night. She has been accusing her daughter of stealing from her, and has also hit her daughter. The patient has fallen twice on her way to the bathroom at night. The daughter has been giving her diphenhydramine for 1 month to help her sleep, but she says it does not seem to be helping. You have also been treating the patient with ranitidine for esophageal reflux and with amitriptyline for depression. Vital signs are normal, and physical examination is unchanged from the last visit. Her mental status has deteriorated from her last visit 4 months ago. Today she is oriented only to name, does not seem to recognize you and appears to be visually hallucinating. Her daughter says she herself is overwhelmed and wonders if it is time to consider a nursing home for the patient. Which of the following is the most appropriate response?

[A) "Her increased confusion may be due to her medicines. Let's explore that possibility first."]
B) "Her worsening mental status may be due to an inadequate diet. Let's explore that first."
C) "It is a hard step to take, but I agree the time has come to arrange nursing home placement."
D) "It is a little premature for that. Let's try some home health services first."
E) "It is possible that medication might make her more manageable. Let's try a course of haloperidol."

2. A 12-year-old boy is brought to the office by his parents for a follow-up visit after starting treatment with carbamazepine 3 months ago for temporal lobe seizures. He has been seizure-free since having attained a therapeutic serum carbamazepine level 2 months ago. He says that he feels well, is doing much better in school, and has no new symptoms or complaints. It is most appropriate to tell him and his parents that with this drug therapy, he will need monitoring to assess for which of the following conditions?

[A) Agranulocytosis]
B) Cardiac arrhythmias
C) Gastric ulcers
D) Proteinuria
E) Renal failure

3. A 72-year-old professor emeritus comes to the office saying, "I am worried that I have Alzheimer's or small strokes or something." During the past year, he has noted increasing difficulty with his memory, especially for names, which has created several awkward moments professionally and socially. He adds, "I'll be doing fine, when all of a sudden my mind goes blank. I can't recall something I should easily know, and then suddenly it will come back to me a couple of minutes later." He complains that he frequently misplaces items like his keys, which is very unlike him. He also complains that his sleep is not restful anymore and that he tosses and turns all night. His wife has told him that she is not sleeping either, because he snores loudly. He says, "I'm tired much of the time, and I doze off whenever I try to read." He also complains of frequent headaches in the morning. Past medical history includes hypertension, which is well-controlled with diltiazem; peptic ulcer disease for which he takes ranitidine; and lumbar osteoarthritis, for which he takes ibuprofen. Height is 175 cm (5 ft 9 in) and weight is 72 kg (160 lb). Vital signs are: temperature 36.9°C (98.4°F), pulse 80/min and regular, respirations 12/min and blood pressure 158/100 mm Hg. Physical examination, including neurologic examination, is normal. He scores 29 out of 30 on the Mini-mental state test. Which of the following is the most appropriate next step?

A) Order CT scan of the head
B) Order electroencephalography
C) Reassure him that his symptoms are probably normal
D) Refer him for neuropsychological testing
[E) Refer him for polysomnography]

4. An 8-year-old boy is brought to the office because of a 5-day history of fever, coryza and cough that coincides with an epidemic of influenza in the community. Today he is unable to walk because of pain in the calves. His mother has been giving him acetaminophen for fever and pain. Physical examination shows a temperature of 38.3°C (101.0°F). He is alert, interactive and well-hydrated. He has clear rhinorrhea, mild pharyngeal erythema and a clear chest. His calves are tender to palpation. Strength cannot be tested because of pain. Neurologic examination, including deep tendon reflexes, is normal. Which of the following is the most appropriate therapy?

[A) Acetaminophen, orally, as needed]
B) Amantadine, orally
C) Immune globulin, intravenously
D) Influenza virus vaccine, intramuscularly
E) Prednisone, orally

5. A 55-year-old Hispanic welder comes to the office for an initial visit because of a lesion in his right eye that has been present for several months. During this time his right and left eyes have been increasingly sensitive to wind. He has not had double vision and has not seen "spots" in his field of vision. He says that he has been generally healthy and has not seen a physician during the past 30 years. He takes no medications. He does not wear corrective lenses. Family history is significant for blindness in his mother at age 77 years. The patient has smoked a half pack of cigarettes daily for the past 40 years, and he drinks an occasional six-pack of beer on weekends. Vital signs today are temperature 37.2°C (99.0°F), pulse 110/min, respirations 20/min, and blood pressure 140/85 mm Hg. Examination of the right eye discloses the finding shown in the photograph. Visual acuity is 20/40 in both eyes. Musculoskeletal examination discloses symmetric enlargement of the proximal and distal interphalangeal joints of both hands. The joints are firm to palpation and cool to the touch. The remainder of the physical examination is noncontributory. Which of the following is the most likely diagnosis?

A) Corneal abrasion
B) Glaucoma
C) Keratitis
[D) Pterygium]
E) Stye

6. A 3-year-old African-American boy who is a new patient is brought to the office by his grandmother. She says, "He was OK until this afternoon, when he suddenly developed a fever. He's been spitting a lot. He keeps his mouth open and he refuses to lie down. He won't eat." You learn that the child has received only one set of vaccinations at the age of 2 months. Vital signs are: temperature 39.4°C (103.0°F), pulse 110/min, respirations 24/min and blood pressure 110/70 mm Hg. On physical examination the child sits in a tripod position and salivation is evident. Which of the following is the most appropriate next step?

A) Administration of cefotaxime, intravenously
B) Complete blood count and blood culture
C) Determination of arterial blood gas values
[D) Immediate otorhinolaryngology consultation]
E) Lateral neck x-ray film

7. A 19-year-old college student comes to the student health center because of palpitations, shortness of breath and a runny nose. He has asthma that he has treated with an over-the-counter cold preparation and an epinephrine metered-dose inhaler every 2 to 3 hours at night. He just used the bronchodilator in the waiting room. Vital signs are: temperature 38.2°C (100.8°F), pulse 82/min and respirations 18/min. He appears to be somewhat anxious and his breathing is labored. Auscultation discloses mildly diminished breath sounds in all lung fields accompanied by scattered wheezing. Which of the following is the most appropriate management?

A) Add oral aminophylline therapy
B) Admit him to the hospital for respiratory therapy
C) Prescribe decongestant/antihistamine therapy
D) Prescribe antihistamine therapy
[E) Substitute an albuterol nebulizer for the epinephrine]

The following vignette applies to the next 2 items.

A 6-month-old African-American girl is brought to the office in January for a well-child visit. She was born at 32 weeks' gestation after a pregnancy complicated by an incompetent cervix and premature labor. She has a 3-year-old brother. Her birth weight was 2700 g (6 lb). At birth, she had mild respiratory distress syndrome and required mechanical ventilation for 36 hours. She also has gastroesophageal reflux disease for which she is given ranitidine, daily. Vaccinations are up-to-date. Developmental milestones are appropriate for her adjusted chronological age. Head circumference and growth charts are shown.

Item 1 of 2

8. Regarding the results on the head circumference chart, which of the following is the most appropriate conclusion?

?A) The growth pattern is most likely due to neonatal intraventricular hemorrhage
B) Her growth is normal for a premature infant
C) An MRI should be done to rule out a brain tumor
D) The parents' head circumferences should be measured to evaluate the infant for familial macrocephaly
[E) She should be examined for possible papilledema to rule out hydrocephalus]

Item 2 of 2

9. Two weeks later the girl is brought back by her mother because of a runny nose and difficulty breathing for the past 3 days. The mother says, "She's up every 2 hours now, and last night she had a fever. I think her brother picked up a cold from preschool and gave it to her. I can't keep him away from her." Vital signs now are temperature 38.5°C (101.3°F), pulse 144/min, and respirations 60/min. On physical examination she is alert but in mild respiratory distress with slight nasal flaring. Auscultation of the chest discloses fine expiratory wheezes bilaterally, and mild intercostal retractions. Which of the following is the most likely cause of the infant's illness?

A) Aspiration pneumonia
B) Asthma
C) Bronchopulmonary dysplasia
D) Mycoplasma pneumoniae
[E) Respiratory syncytial virus]

10. A 72-year-old woman comes to the health center for the first time because of palpitations for the past 3 weeks. She says she has felt tense and has had trouble sleeping.Vital signs are: temperature 37.0°C (98.6°F), pulse 104/min, respirations 18/min and blood pressure 142/80 mm Hg. Physical examination is normal except for a mild bilateral hand tremor. Electrocardiogram shows sinus tachycardia but is otherwise normal. Which of the following diagnostic studies will most likely rule out an organic cause for her symptoms?
A) 2-Hour postprandial serum glucose concentration
B) Echocardiography
C) Holter monitoring
[D) Serum thyroid-stimulating hormone concentration]
E) Toxicologic screening of the urine

11. A 48-year-old man who smokes cigarettes has had progressive claudication in the left calf for the past month. He says the pain lasts about 5 minutes and then subsides. Physical examination shows absent pulses in the left foot and normal pulses in the right foot. Atrophic changes are noted in both legs. Doppler examination shows a 0.40 left ankle/brachial ratio; there is no change with exercise. Which of the following is the most likely diagnosis?

A) Aortic occlusive disease
B) Femoral popliteal occlusive disease
C) Leriche syndrome
D) Peripheral small-vessel occlusive disease
[E) Thromboangiitis obliterans]

12. A 52-year-old Hispanic computer technician comes to the office because of a 3-week history of substernal chest discomfort when she climbs stairs or eats a heavy meal. She first noticed the discomfort after climbing two flights of stairs. The discomfort is nonradiating and sometimes only involves the left side of the chest. She has hypertension and type 2 diabetes mellitus. Current medications include metformin and an ACE inhibitor. She has recently had increased stress because her company is experiencing financial difficulty. Her husband receives medical disability benefits, and they are dependent on her income. Physical examination shows no abnormalities. Which of the following risk factors is most important to consider when assessing her chest pain?

A) Age
[B) Diabetes mellitus]
C) Gender
D) Hypertension
E) Stress level

13. A 47-year-old woman returns to the office because of gastrointestinal symptoms. She says, "I still have burning pain in my stomach that travels up my chest to my neck after I eat." During the past 5 years she has been treated with antacids, H2-blocking medications, proton pump inhibitors and motility agents, with only mild relief. She smokes one pack of cigarettes per day and drinks one cup of coffee in the morning. There is no family history of peptic ulcer disease. Previous endoscopies, the last of which was 6 months ago, have shown lower esophagitis secondary to reflux with healing ulcers and scarring. Gastric and duodenal cultures for Helicobacter pylori have been negative. Vital signs today are normal. Physical examination, including rectal examination, is normal. Which of the following is the most appropriate next step?
A) Consider an alternative pharmacotherapeutic regimen
B) Continue current treatment
C) Do esophageal pH monitoring
[D) Obtain surgical consultation]
E) Repeat endoscopy

14. A 38-year-old woman with systemic lupus erythematosus but no evidence of nephritis comes to the office because of a 3-week history of mood swings with crying spells, irritability and insomnia. She is especially upset because she has been yelling at her children "over small, everyday things." One month ago she started corticosteroid therapy. She is currently taking prednisone, 60 mg/day. Her other medications include an oral contraceptive (the same one for the past 6 years) and ibuprofen. She smokes one-half pack of cigarettes daily, drinks one to two beers 5 nights weekly and three cups of coffee each morning. Which of the following is the most appropriate intervention for her mood disturbance?

A) Add amitriptyline at bedtime
B) Discontinue the oral contraceptive
?C) Reduce the prednisone dosage
D) Replace ibuprofen with acetaminophen
[E) Urge her to stop smoking and to reduce her alcohol and caffeine intake]

15. A 76-year-old retired pharmacist is brought to the health center by his wife, who says, "He's afraid to go to sleep, Doctor. Tell him, Henry." He tells you that he was mugged and assaulted 1 week ago while he was out for a walk early in the morning. The patient proceeds to tell you that he has been having nightmares, not about the assault, but of being in vulnerable situations. He also feels anxious during the day but he is able to leave the house without difficulty. In addition to supportive therapy, which of the following pharmacotherapies is most appropriate to prescribe?

A) Amitriptyline
[B) Clonazepam]
C) Diphenhydramine
D) Gabapentin
E) Risperidone

16. A 58-year-old white store manager comes to the office for a periodic health evaluation. You have been treating both the patient and his wife for the past 15 years. Today the patient is tearful and agitated. He says that he is having difficulty with his son, who is age 32 years and has schizophrenia. The son has been living intermittently in a group home or on the streets. The patient says that his son is noncompliant with his antipsychotic medications. Recently, the son has been calling the patient's house asking for money, which the patient suspects his son uses to buy alcohol and illicit drugs. It is most appropriate to advise the patient to do which of the following?

?A) Arrange an involuntary commitment to a psychiatric hospital for his son
B) Ask his son's psychiatrist to adjust his medication
C) Change his phone number
[D) Contact the local chapter of the National Alliance for the Mentally Ill for support and advice]
E) Obtain a restraining order against his son

17. A 20-year-old man comes to the health center because of ankle pain. Two days ago he sustained an inversion injury of his left ankle in a basketball game. He has been able to walk unassisted since the injury. Today he has pain and moderate swelling and discoloration over the lateral malleolus. Physical examination shows tenderness on palpation over the anterolateral corner of the ankle joint. He has had two similar injuries in the past. Which of the following is the most appropriate initial management?

A) An ankle-strengthening exercise program
B) Application of a long-leg cast for 3 weeks
C) Application of a short-leg cast for 3 weeks
D) Protected weight bearing
[E) Surgical repair of the ankle ligaments]

18. A 52-year-old woman comes to the office because of a 4-day history of increasing pain of the right hip and thigh. The pain is exacerbated by lying on her right side while sleeping. She says the pain often awakens her and is accompanied by a burning sensation along the right side of her posterior thigh that radiates to her knee. She usually has stiffness and pain in the hip during the following morning that gradually diminishes as she walks around her house and does house chores. She says the pain is also triggered by sitting with her right leg crossed over the left leg. The patient is otherwise healthy and takes no medications. She is 168 cm (5 ft 6 in) tall and weighs 63 kg (140 lb); BMI is 23 kg/m2. Vital signs are normal. Physical examination discloses tenderness on deep palpation of the right trochanter. Which of the following is the most likely diagnosis?

[A) Arthritis of the hip]
B) Aseptic necrosis of the femoral head
C) Bursitis
D) Gout
E) Osteosarcoma of the femoral head

19. An 18-year-old man comes to the health center because he has had pain in his right leg for the past 5 days. He says that he recently added jogging to his weight-lifting workouts, and he started running 5 miles per day 2 weeks ago. He is 180 cm (5 ft 11 in) tall and weighs 83 kg (185 lb). On physical examination he has moderate tenderness over the midtibia. X-ray of the leg will most likely show which of the following?

A) A bone cyst
B) Displaced fracture
C) Metastatic disease
D) Soft-tissue calcification
[E) Normal findings]

20. A 58-year-old woman comes to the office for follow-up of fibromyalgia. You had been treating her for the past several years for nonspecific muscular aches and pains. In the past you noted that occasionally the pain could be reproduced on physical examination by applying pressure to certain muscles; however, these trigger points seemed to change on each physical examination. Fibromyalgia was diagnosed 3 months ago and amitriptyline therapy was started at that time. At a follow-up visit 3 weeks ago she showed little response to the amitriptyline therapy, and naproxen was added to her regimen. Today she returns to the office complaining that "every time I get the least little bump on my hands the skin seems to tear." Her physical examination is unchanged except for the lesions shown. Which of the following is the most appropriate management?

A) Discontinue the amitriptyline
[B) Discontinue the naproxen]
C) Prescribe oral corticosteroids
D) Prescribe topical corticosteroids
E) Prescribe topical 5-fluorouracil

21. A 10-year-old Asian girl is brought to the office by her mother because of a painful swelling in the girl's neck. The child says that for the past 4 days the right side of her neck has been sore. Today she showed it to her mother, who noticed a red swelling. She does not have fever or chills. She has not missed any days of school. She lives with her parents and two siblings in a suburban community. Further discussion discloses that the family recently adopted an 8-week old kitten from the animal shelter. Vital signs now are: temperature 37.2°C (98.9°F), pulse 80/min, and respirations 24/min. On physical examination, the girl has a 2H4-cm, red, tender mass with overlying erythema and induration in the right submandibular area. There is a crusted papule on the right cheek. Which of the following is the most likely cause of this patient's condition?

[A) Bartonella henselae]
B) Branchial cleft cyst
C) Epstein-Barr virus
D) Trauma
E) Untreated impetigo

22. A 64-year-old retired teacher comes to the office with her husband. She has felt fatigued for several months and feels she is losing her memory. She says, "I feel slowed and can't remember what happened yesterday." Her husband reaffirms this history and notes that she recently got lost in a local mall and called him at home to come and get her. She was frightened by this episode and had insomnia that night, even though she otherwise has been sleeping longer than usual. She has a good appetite. There is a family history of senile dementia in her father and two uncles. Her only medication is daily conjugated estrogen. Her height is 163 cm (5 ft 4 in) and her weight is 64 kg (142 lb), which is an increase of 2.5 kg (6 lb) since you saw her 8 months ago. Pulse is 54/min and regular, and blood pressure is 140/86 mm Hg. On physical examination she appears pale. Deep tendon reflexes have a slow relaxation phase; the remainder of the examination is normal. A Mini-mental state test shows a delayed recall of one of three items and failure on serial 7s. She cannot recall any past presidents but she knows the current president. Based on these findings, which of the following is the most likely working diagnosis?

A) Early dementia, Alzheimer type
[B) Hypothyroidism]
C) Pernicious anemia
D) A transient ischemic attack
E) Vascular dementia

23. A 38-year-old obese woman with a 2-year history of type 2 diabetes mellitus has not lost weight despite persistent advice concerning diet and exercise. She has also been irregular in keeping appointments and has failed to take her medication on a regular basis. There is increasing concern that serious complications will occur unless she becomes more cooperative. You decide to terminate the physician-patient relationship, hoping that another physician can establish better rapport and help her with her problems. The most appropriate way to terminate the relationship is to do which of the following?

A) Give her a copy of her medical records at the next appointment and advise her to find another physician
B) Give her a list of three physicians and ask her to pick one, to whom you will send her medical records
C) Refuse further treatment unless she loses 2.7 kg (6 lb) before her next appointment in 1 month
D) Tell her 19-year-old daughter, who brings her to the office, to convince her mother to find another physician and tell her the reasons for this suggestion
[E) Tell her that the relationship will be terminated in 1 month, give her reasons for this decision, and offer her a list of three physicians' names]

24. A 56-year-old Native American man returns to the office to discuss results of studies obtained during a previous visit 8 weeks ago. The patient has a 19-year history of diabetes mellitus treated with sulfonylurea. He checks his serum glucose concentration approximately once daily. He does not smoke cigarettes and rarely drinks alcoholic beverages. He is 183 cm (6 ft) tall and weighs 76 kg (168 lb); BMI is 23 kg/m2. Vital signs during the previous visit were temperature 36.9°C (98.4°F), pulse 82/min, and blood pressure 130/85 mm Hg. Physical examination of the neck disclosed a right-sided carotid bruit. Examination of the extremities disclosed diminished pulses with associated hair loss over both legs. Hemoglobin A1c was 7.2%. Urine albumin-creatine ratio was 62 mg/g/24 h (N
Reply
#5
Ankle-brachial index (ABI) was 0.89 on the left and 0.98 on the right; duplex carotid ultrasonography showed nonulcerated plaque with 70% stenosis in the right internal carotid artery. The patient was referred to an ophthalmologist, who diagnosed him with nonproliferative background diabetic retinopathy. Which of the following findings in this patient is of most concern?
[A) 70% stenosis of the right carotid artery]
B) Hemoglobin A1c of 7.2%
C) Left ABI of 0.89
D) Nonproliferative diabetic retinopathy
E) Urine albumin-creatine ratio of 62 mg/g/24 h


25. A 25-year-old woman returns to the office because of intermenstrual spotting since beginning oral contraceptive therapy 6 months ago. This is her first attempt at oral contraceptive therapy; she and her partner primarily used condoms for contraception in the past. Before this current therapy her menstrual periods had always been regular. Which of the following is the most appropriate management?

A) Advise her to take two pills daily until the bleeding stops
B) Discontinue the oral contraceptive therapy and have her resume use of condoms for birth control
C) Reassure her that the bleeding problem will resolve in a few months
[D) Switch the current oral contraceptive pill to one containing a higher estrogen dose]
E) Switch to a progestin-only oral contraceptive pill

The following vignette applies to the next 2 items.


A 17-year-old girl is brought to the health center by her mother because the girl has had only two menstrual periods in the past 8 months. She had regular menstrual periods from menarche at age 14 years until 18 months ago. For the past 18 months she has been training as a long-distance runner.

Item 1 of 2

26. The underlying mechanism of her menstrual dysfunction is best defined by an abnormality in which of the following serum concentrations?
A) Androstenedione
B) Creatine kinase
[C) Luteinizing hormone]
D) Testosterone
E) Thyroid-stimulating hormone (TSH)

Item 2 of 2

27. Which of the following is the most likely cause of the menstrual irregularity?
A) High-carbohydrate diet
B) Increased basal body temperature
C) Increased muscle mass
[D) Loss of body fat]
E) Repeated volume depletion


28. A 23-year-old white nulligravid woman returns to the office for follow-up of a 2-year history of primary infertility. Menstrual periods occur at regular 28-day intervals. She has a history of chronic pelvic pain. Analysis of her husband's semen shows a sperm count of 40 million with 65% motility and normal morphology. Diagnostic laparoscopy with hydrotubation shows normal pelvic anatomy, with no evidence of endometriosis or pelvic adhesions. Both fallopian tubes spill methylene blue dye. The patient's basal body temperatures recorded during the past month are shown. Which of the following is the most likely cause of the patient's inability to conceive?

?A) Anovulation
B) Male factor infertility
C) Old pelvic inflammatory disease
D) Polycystic ovary syndrome
[E) No cause can be identified at this time]

29. In the course of routine prenatal care at the health center, a 30-year-old Hispanic woman is found to have gestational diabetes. This is her first pregnancy and she is at 20 weeks' gestation. She should be counseled that adequate prenatal care, including regular determination of blood glucose concentrations and adherence to a controlled dietary regimen, may reduce the possibility of which of the following?

[A) The fetus developing macrosomia]
B) Her developing essential hypertension
C) Her developing type 1 diabetes mellitus in the future
D) Intrauterine growth restriction
E) Premature delivery

30. A 20-year-old woman returns to the office for the results of her prenatal laboratory studies. By date of her last menstrual period she is 14 weeks pregnant with her second child. Her first pregnancy, which you followed, resulted in an uncomplicated vaginal delivery approximately 11 months ago. Her prenatal serology screening for syphilis is positive with a titer of 1:126, and a fluorescent treponemal antibody absorption (FTA-ABS) test is positive. The patient denies having lesions consistent with syphilis, and her current physical examination shows no lesions. Serology during her first pregnancy was negative. At this time, which of the following is the most appropriate management?

A) Do an amniocentesis to obtain fluid for darkfield evaluation
[B) Follow the treatment regimen recommended for primary or secondary syphilis]
C) Postpone treatment until the patient is at least 20 weeks pregnant
D) Schedule a lumbar puncture before instituting treatment
E) Withhold treatment until further studies rule out a biologic false-positive

31. A 34-year-old man comes to the office with his wife and daughter because he has had some dusky lesions on his shoulder for the past 2 months. He says that two have become larger during the past week. He and his wife have recently adopted an 18-month-old girl from the Ukraine who has a similar rash. He is an environmental scientist and his work requires him to travel overseas and work outdoors. He is concerned about the possibility of skin cancer. Vital signs are normal. Physical examination shows a cluster of six discrete papular lesions on his left shoulder that are slightly tender. These lesions have a central depression containing some pus-like material. There is no axillary or cervical adenopathy. His rash is shown. Which of the following is the most appropriate management for the patient?
?A) Acyclovir
B) Scabicidal cream
C) Topical corticosteroid cream
D) Topical fluconazole
[E) Topical liquid nitrogen]

32. A 25-year-old Latino man comes to the health center for a periodic health evaluation. He tells you that he has attended a day-treatment program for his schizophrenia, paranoid type, since his discharge from the hospital 1 year ago. The patient's most recent psychiatrist is moving away and he now wants you to refill his medications. He takes haloperidol, benztropine and valproic acid. He says, "The voices aren't telling me to harm myself anymore. And I know now that my food is not poisoned." The patient is 183 cm (6 ft) tall and weighs 86 kg (190 lb). Vital signs are: temperature 37.0°C (98.6°F), pulse 72/min, respirations 14/min and blood pressure 130/86 mm Hg. Physical examination is significant for darting and protruding movements of the tongue and some facial grimacing. Which of the following is the most appropriate change in pharmacotherapy?

A) Prescribe vitamin A
B) Increase the dose of benztropine
C) Increase the dose of haloperidol
[D) Replace haloperidol with risperidone]
E) Replace valproic acid with lithium

33. A third-year medical student returns to the student health service for the third time because he thinks he has ulcerative colitis. After a thorough history and physical examination, he is told that no organic disease is present. Despite that reassurance, the student continues to test his stool for blood and continues to believe that his physicians have missed the correct diagnosis. This behavior is most characteristic of which of the following?

A) Conversion disorder
B) Depersonalization
[C) Hypochondriasis]
D) Munchausen syndrome
E) Somatization disorder

34. A 10-year-old Asian-American boy is brought to the office for a sports physical examination. He has been healthy except for a few episodes of otitis media as an infant. He has had no shortness of breath, syncope or chest pain in the past. Growth has been normal. Vital signs are: temperature 36.6°C (97.8°F), pulse 80/min, respirations 16/min, and blood pressure 110/76 mm Hg. Cardiac examination discloses an early systolic click at the apex, a midsystolic ejection murmur at the right upper sternal border and a thrill in the suprasternal notch. The remainder of the physical examination is normal. The patient is referred to the cardiologist and the diagnosis of aortic stenosis is confirmed by echocardiogram. Close follow-up and graded exercise testing are recommended. The patient is at increased risk for which of the following?

A) Atrial arrhythmias
B) Complete heart block
C) Coronary artery disease
D) Pulmonary hypertension
[E) Sudden death]

35. A 12-year-old Haitian boy is brought to the health center for the first time by his mother and maternal grandmother. The mother states, "There was a voodoo curse placed on my family and now my son is having problems because of that." She reports that for the past 6 months he has been talking back to his teachers, has been suspended from school for skipping class, and has been defiant with all adults. He is failing two subjects in school and will need to attend summer classes. His medical history is significant for an allergy to sulfa drugs, a positive PPD skin test for which he was treated prophylactically at age 10 years, and encopresis that resolved. He currently takes no medications. The patient is 163 cm (5 ft 4 in) tall and weighs 49 kg (108 lb). Vital signs are: temperature 37.0°C (98.6°F), pulse 80/min, respirations 18/min and blood pressure 90/50 mm Hg. Which of the following is the most appropriate opening statement to the mother?

A) "Do you think your son may be taking drugs?"
B) "I suggest you consult a voodoo priest for help with this problem."
[C) "Tell me more about the voodoo and its effect on your son's behavior."]
D) "There is no such thing as voodoo."
E) "Voodoo does not cause children to have bad behavior."

36. You are invited to a council meeting of Native-Americans to discuss a heptavalent pneumococcal polysaccharide-protein conjugate vaccine (Prevnar7) for use in the community's children, many of whom are younger than age 12 months. All of the infants and children in the community are up-to-date on standard recommended vaccinations. The council spokesperson says, "We are skeptical about the safety of giving our sons and daughters yet another vaccine." Which of the following is the most compelling reason to recommend vaccination of all infants younger than 12 months of age?
A) Administration of the vaccine will decrease the incidence of invasive pneumococcal disease among the children in the community
B) Administration of the vaccine will decrease the likelihood of secondary pneumonia during respiratory syncytial virus (RSV) season
[C) Administration of the vaccine will significantly decrease the severity of acute otitis media and prevent hearing loss]
D) Native-American children make ineffective antibodies when the 23-valent pneumococcal vaccine alone is administered to them
E) The peak incidence of pneumococcal disease occurs in children younger than 12 months of age

Reply
#6
Block 2
1. A 76-year-old woman is admitted to the hospital after a fall at her home earlier in the day. She has been followed in your practice for several years and is in generally good health. She drinks socially, does not smoke and has been active in senior citizen groups. She takes ibuprofen occasionally for pain, but no other medications. On admission she is alert and oriented, and complains only of pain in her left leg. Her temperature is 36.1°C (97.0°F), pulse is 72/min and regular, and blood pressure is 140/85 mm Hg. X-ray films taken upon arrival show a fracture of the left femoral shaft. Surgical repair of the fracture is done the next morning. The patient receives 2 U of packed erythrocytes during the procedure. Postoperative medications include morphine, prophylactic cephalothin and low-dose warfarin. On the evening of the operation, the patient becomes combative, begins to hallucinate and has a brief, generalized seizure. A fine petechial rash is noted on her chest. Which of the following is the most likely cause of her seizure?
A) Antibiotic allergy
B) Blood transfusion reaction
[C) Fat emboli]
D) Hemorrhagic stroke
E) Unrecognized cerebral concussion

2. A 72-year-old woman with metastatic rectal cancer is admitted to the hospital because of weakness and altered mental status. She has bilateral ureteral stents due to prior obstruction from the cancer. She has been using a fentanyl patch for several weeks. Physical examination on admission is notable for lethargy and 3-mm reactive pupils but no focal neurologic signs. She is afebrile and has a blood pressure of 120/82 mm Hg. Laboratory studies show serum urea nitrogen (BUN) concentration of 60 mg/dL and a serum creatinine concentration of 6.2 mg/dL. Serum bilirubin concentration is normal. One week ago serum BUN concentration was 30 mg/dL and serum creatinine concentration was 3.0 mg/dL. Which of the following is the most likely explanation for her altered mental status?

A) Fentanyl intoxication
B) Hepatic encephalopathy
C) Hypercalcemia
[D) Uremia]
E) Urosepsis

3. A 54-year-old African American woman has been in the intensive care unit (ICU) for the past 10 hours because she has failed to regain consciousness after passing out at a restaurant 11 hours ago. Upon initial arrival at the emergency department the patient's friend stated that the patient had remarked about the sudden onset of a terrible headache and neck stiffness while they were having lunch. A few minutes after the onset of the headache, she became confused, vomited, and lost consciousness. She regained consciousness briefly en route to the hospital, but she has been unconscious since admission. On arrival she was intubated and mechanically ventilated and transferred to the ICU, where intravenous fluids were started. Vital signs are: pulse 110/min and blood pressure 174/96 mm Hg. She exhibits no spontaneous movement and is unresponsive to verbal or painful stimuli. Lung fields are clear to auscultation. CT angiogram confirms subarachnoid hemorrhage from a ruptured cerebral aneurysm and impaired intracranial circulation. The family should be counseled regarding which of the following?

A) The futility of continued life support
B) The need for screening of first-degree relatives for aneurysms
C) The need for them to sustain hope
[D) The need to have a guardian ad litem appointed to make informed decisions about the patient's care]
E) The probability that the patient will not regain consciousness

4. A previously healthy 3-year-old boy is admitted to the hospital through the emergency department because he has been having generalized seizures. He had a fever this morning and had one seizure at home and another on the way to the emergency department. While in the emergency department he had several additional seizures that were controlled with intravenous lorazepam. Vital signs on arrival at the emergency department were temperature 39.6°C (103.3°F), pulse 110/min, respirations 24/min, and blood pressure 85/60 mm Hg. Physical examination showed no abnormalities except for left otitis media and lethargy. Intravenous phenytoin was administered. Spinal fluid examination, serum electrolyte concentrations and complete blood count were normal. Now in the hospital, the patient has another seizure. The patient is managed appropriately and the seizure stops. When discussing this child's management and prognosis with the parents, it is most appropriate to counsel them about which of the following?
A) Their child has an increased risk for attention-deficit/hyperactivity disorder
B) Their child has an increased risk for developing a brain tumor
C) Their child is likely to be developmentally delayed
D) Their child will have a seizure every time he has a fever
[E) Their child will probably become seizure-free as he becomes older]

5. A 21-year-old Asian-American man who sustained a closed head injury in a motorcycle accident is admitted to the hospital. He is intubated and is receiving mechanical ventilation in the intensive care unit. On repeat physical examinations during the next 2 days, he shows no clinical signs of cerebral activity and has no spontaneous respiration. His pupils are fixed in mid-position and he has no vestibulo-ocular reflexes. Electroencephalograms obtained 24 hours apart have nearly flat tracings. His driver's license indicates his desire to be an organ donor. When you inform his parents that he is brain dead, you remind them of his donor statement and request permission to harvest usable organs. The parents are shocked; they say they are unaware of his desire to be an organ donor and refuse to grant permission. Because the patient meets the clinical criteria for brain death, which of the following is most appropriate?
[A) Explain how their son's death can contribute to another person's life]
B) Have a potential organ recipient talk with the family
C) Obtain a court order authorizing removal of usable organs
D) Order cerebral angiography to convince the parents that their son is brain dead
?E) Proceed with organ removal because the patient is an adult

6. A 52-year-old woman is admitted to the hospital for diagnostic evaluation of a slight aphasia and change in her personality, as noted by her husband. CT scan of the head discloses a 3-cm mass in the left temporal lobe and a 5-cm mass in the right frontal lobe. Because she is a chronic cigarette smoker, you have been obtaining annual chest x-ray films, which have been reported as normal. Her chest x-ray film on admission today is reported as showing a "2-cm spiculated mass in the left subapical region." No reference is made to comparing this film with the prior films. The radiologist then calls you and admits to you that the lesion was present on her prior films but it was overlooked. Which of the following is the most appropriate next step?

A) Agree with the radiologist that informing the patient and her family will do more harm than good
B) Ask the radiologist to inform the patient and her family of the problem
C) Compliment the radiologist for finding what will probably be the primary lesion accounting for the two CNS lesions
D) Consult with your attorney regarding avoidance of litigation for yourself
[E) Inform the radiologist of your responsibility to inform the patient and her family of the problem]

The following vignette applies to the next 2 items.


While making rounds in a rehabilitation facility, you see a 76-year-old white woman who has been your patient for 25 years. One month ago she was diagnosed with metastatic non-small cell carcinoma of the lung with metastases to the other lung, liver and brain. She now has moderate left hemiparesis that limits walking. She also has multiple, asymptomatic, deep venous thrombi of both legs that are being treated with a vena caval filter. Despite discussion with the patient and her family about her illness, her son tells you that she is apparently unaware that her condition is terminal.

Item 1 of 2

7. Which of the following is the most appropriate next step?
A) Arranging transfer to the in-patient psychiatric ward
B) Beginning anxiolytic therapy
[C) Discussing the prognosis again with the patient]
D) Encouraging the family to discuss the patient's prognosis with her
E) Explaining to the son that it is inappropriate for you to speak about his mother's illness

Item 2 of 2

8. The patient is not in severe pain and she currently has no respiratory distress. She has chosen not to have further intervention and she probably has about 3 months to live. Her husband is an active 75-year-old man with recently diagnosed type 2 diabetes mellitus. Two of their three sons live 1 hour away. The patient has many concerned neighbors and friends. The patient's family asks for your advice regarding appropriate care for her for the remainder of her illness.


Which of the following is the most appropriate recommendation?

A) Continued care in the rehabilitation center until she is able to ambulate on her own
B) Home health care for as long as she qualifies for skilled nursing services
[C) Home hospice care with physical therapy]
D) Placement under intermediate (nonskilled) care in a local nursing home
E) Placement under skilled care in a local nursing home

9. An 18-year-old woman gave birth to a healthy neonate 72 hours ago. The woman now has the acute onset of tachypnea. Vital signs are normal. Diffuse wheezing is noted on auscultation of the chest. Chest x-ray film is normal. Electrocardiogram shows evidence of right heart strain. Which of the following is the most likely diagnosis?

A) Adult respiratory distress syndrome
B) Aspiration pneumonia
C) Bronchopneumonia
D) Congestive heart failure
[E) Pulmonary embolism]

10. A 34-year-old woman undergoes a middle ear operation with oral intubation and general anesthesia. During the 4-hour procedure she was in a semi-lateral position with her head maximally turned to the right and her chin tucked on her chest. She awakens normally at the end of the procedure and is extubated without difficulty. Approximately 30 minutes later she appears anxious and is sitting upright. She leans forward and says, "I'm not getting enough air." Her voice is barely audible and very hoarse and she appears to be struggling to take a deep breath. Vital signs are: pulse 94/min, respirations 28/min and blood pressure 128/72 mm Hg. Oxygen saturation is 92% with an FIO2 of 0.4. Oxygen flow is maximally increased and O2 saturation is 93%. Auscultation of the chest discloses bilateral breath sounds with crowing sounds on inspiration and expiration. Which of the following is the most appropriate pharmacotherapy?

A) Midazolam
B) Naloxone
[C) Nebulized racemic epinephrine]
D) Neostigmine
E) Succinylcholine

11. A 67-year-old man with a history of poorly controlled hypertension is admitted to the hospital because of increasing fatigue, weight loss and dyspnea on exertion for the past 6 weeks. Medical history includes tonsillectomy, adenoidectomy, mitral valve commissurotomy and a sigmoid colon resection for diverticulitis. Vital signs are: pulse 90/min, respirations 14/min and blood pressure 190/100 mm Hg. The patient appears chronically ill but is in no acute distress. Laboratory studies show:
Serum
Urine
Na+
140 mEq/L
WBC
0/hpf
K+
4.5 mEq/L
RBC
4/hpf
Cl-
100 mEq/L
HCO3-
25 mEq/L
Which of the following is the most likely finding on echocardiography?

A) Ejection fraction of 60%
[B) Left ventricular wall thickening]
C) Pericardial fluid
D) Septal dyskinesia
E) Thoracic aortic aneurysm

the following vignette applies to the next 2 items.

You have been treating a 5-month-old child in the neonatal intensive care unit (NICU). He was delivered at 26 weeks' gestation by cesarean delivery because of premature rupture of membranes. The mother is 18 years old and is unemployed. There are two other children in the home. The father is not living with them, and he has not been in contact with the mother. The mother rarely visits the NICU. The infant had severe respiratory distress syndrome at birth and required dopamine for blood pressure support. His condition progressed to pulmonary interstitial emphysema and bronchopulmonary dysplasia by 4 weeks of age. At 4 months of age he required a tracheostomy and medication for control of his blood pressure. Now, at 5 months of age, the boy has a cardiac arrest requiring resuscitation and placement of chest tubes. He then develops seizures that are eventually controlled with medication. The neonatologist feels that the child will always require life support. The mother refuses to discuss the possibility of withdrawing life support. Her insurance is Medicaid. The hospital bill for this child is now $350,000.

Item 1 of 2

12. To guide further treatment at this time, it is most important to assess which of the following?
[A) The extent of neurologic injury]
B) The financial impact on the family

C) The mother's level of understanding of the child's prognosis
D) Whether another family member is better suited to make decisions about the patient's care
E) Your state's law regarding futile care

Item 2 of 2

13. One week later the mother and the attending physician are unable to reach an agreement on the management plan. Which of the following is the most appropriate next step?

A) Ask the court to appoint a guardian
B) Defer major decisions until the father can be located
[C) Have the social worker contact Medicaid regarding further coverage]
D) Involve the hospital bioethics committee

E) Maintain the patient on a respirator but transfer him out of the NICU
14. A 42-year-old African-American man with hepatitis C is admitted to the hospital for evaluation of a 3-day history of fever and right upper quadrant abdominal pain. The pain is steady and does not change with position. He has had associated nausea but has not vomited. He has a history of alcohol dependence but has not had an alcoholic beverage since being diagnosed with hepatitis C 4 years ago. He does not take any medications. On admission, vital signs are: temperature 38.5°C (101.3°F), pulse 115/min, respirations 24/min and blood pressure 100/60 mm Hg. Physical examination shows scleral icterus. Abdominal examination discloses mild distention and right upper quadrant tenderness to palpation; the liver spans 25 cm. There is no rebound tenderness and bowel sounds are normal. Ultrasonography of the right upper quadrant shows three small gallstones, dilated right and left intrahepatic ducts and normal common bile duct. Laboratory studies show:

Serum
Blood
ALT
650 U/L
Hematocrit
35%
AST
442 U/L
WBC
22,000/mm3
Alkaline phosphatase
342 U/L
PT
14 sec
Bilirubin
7.0 mg/dL
PTT
28 sec
Na+
140 mEq/L
INR
1.2
K+
4.2 mEq/L
Cl-
104 mEq/L
HCO3-
23 mEq/L

Fluid resuscitation is begun. Which of the following is the most appropriate next step?

A) Biopsy of the liver
?B) Cholecystectomy
C) CT scan of the abdomen
[D) Endoscopic retrograde cholangiopancreatography with st]

15. A 16-year-old Mexican American girl who is 28 weeks pregnant comes to the health center with her boyfriend because of epigastric pain radiating to her back. She says, "I have had pain for many days, especially after eating." She has been taking acetaminophen for the pain several times a day for the past week. She admits to drinking an occasional bottle of light beer. She has been receiving prenatal care at the health center. Vital signs today are temperature 37.5°C (99.5°F), pulse 110/min, respirations 15/min, and blood pressure 110/70 mm Hg. Height is 157 cm (5 ft 2 in) and weight is 90 kg (200 lb). Physical examination shows scleral icterus and epigastric and right upper quadrant tenderness. Uterus is appropriately enlarged for her gestational stage. The patient is admitted to the hospital for observation. Results of laboratory studies are shown:
Serum
ALT
55 U/L
AST
58 U/L
Alkaline phosphatase
350 U/L
Amylase
3000 U/L
Bilirubin
Total
7.3 mg/dL
Direct
5.9 mg/dL
Lipase
475 U/L (N=10140)

These laboratory results are most indicative of which of the following?

A) Acetaminophen-induced hepatic necrosis
B) Alcoholic pancreatitis
[C) Biliary pancreatitis]
D) Hepatitis C infection
E) Mononucleosis

16. A 22-year-old woman is in her 5th hospital day following admission for evaluation of severe right lower quadrant abdominal pain. During the previous 8 months, she has had episodes of abdominal pain with associated recurrent diarrhea, which sometimes contained blood. The pain and diarrhea became increasingly severe during the 5 days prior to admission. During that time, she lost 2 kg (5 lb) and had a temperature to 39.0°C (102.0°F). Abdominal examination on admission disclosed a right lower quadrant mass. Blood hemoglobin concentration was 15.5 g/dL and leukocyte count was 24,000/mm3. Colonoscopy showed linear ulcerations with discontinuous areas of inflammation. Biopsy of the lower intestine disclosed noncaseating granulomas. Serum antibody test to Saccharomyces cerevisiae was positive. Treatment with intravenous hydrocortisone therapy was begun. Today, the patient's abdominal pain and diarrhea are resolved. Vital signs are temperature 37.0°C (98.6°F), pulse 76/min, respirations 12/min, and blood pressure 122/74 mm Hg. The abdomen is soft and nontender to palpation; bowel sounds are normal. Which of the following is the most appropriate pharmacotherapy for this patient?

A) Azathioprine
[B) Mesalamine]
C) Metronidazole
D) Prednisone
E) Ranitidine

The following vignette applies to the next 2 items.


A 28-year-old man who lives in a small rural township is admitted to the hospital because of gastrointestinal bleeding. On admission the patient is pale and has orthostatic hypotension. Hemoglobin concentration is 8 g/dL. His condition is stabilized. Emergent upper endoscopy shows a visible gastric vessel, which is treated appropriately.

Item 1 of 2

17. Specific additional history should be obtained regarding which of the following?

?A) Consumption of alcoholic beverages

B) Family history of cancer

C) Source of drinking water

D) Travel history

[E) Use of tobacco products]

Item 2 of 2

18. Which of the following is the most appropriate laboratory study?

[A) Antibodies for Helicobacter pylori]
B) Vitamin B12 (cyanocobalamin) concentration
?C) Ferritin concentration
D) Gastrin concentration
E) Iron concentration


19. A 74-year-old woman is admitted to the hospital for treatment of a hip fracture. Her only medical problem is chronic insomnia; she told the nurse, "I have taken sleeping pills for years." Two days after the operation she has a tonic clonic seizure. Vital signs now are: temperature 38.0°C (100.4°F), pulse 110/min and blood pressure 180/100 mm Hg. She is confused, has diaphoresis and is trembling. Which of the following is the most likely diagnosis?

[A) Barbiturate withdrawal]
B) Diphenhydramine withdrawal
C) Propoxyphene withdrawal
D) Pulmonary embolus
E) Stroke

20. You have just returned from a trip out of town and learn that your long-time patient with end-stage chronic obstructive pulmonary disease has been hospitalized and is on a ventilator. On the way to the intensive care unit to visit him, you are informed that the patient has just died. On your way into the patient's room you meet the patient's 68-year-old wife. Which of the following is most appropriate to say to her?

A) "How do you feel?"
[B) "I am sorry for your loss."]
C) "I know you are upset."
D) "What happened while I was gone?"
E) "Your husband didn't suffer."

21. A 3-day-old Latina infant is in the hospital awaiting routine physical examination. She was born to a 28-year-old woman by cesarean delivery at term because of breech presentation. She is the woman's first child and the pregnancy was uncomplicated. The infant weighed 3686 g (8 lb 2 oz) at birth. Physical examination discloses asymmetry in the number of thigh skin folds and uneven knee levels when the hips and knees are flexed. Barlow test is done and a palpable "click" is felt in the left hip. Without prompt treatment, this child is at increased risk for which of the following?

[A) Dysplasia of the hip]
B) Femoral head anteversion
C) Legg-Calve-Perthe disease
D) Metatarsus adductus
E) Slipped capital femoral epiphysis

The following vignette applies to the next 2 items.


An 83-year-old woman is admitted to a nursing home for ongoing management of Parkinson disease. Her husband can no longer care for her at home because she is moderately demented, has difficulty walking, has fallen several times and has urinary incontinence. She has difficulty swallowing her pills, and she needs assistance with feeding. Despite her husband's attentive care, she has lost 9 kg (20 lb), going from 47.1 kg (104 lb) to 38 kg (84 lb) during the past 9 months. After 2 weeks in the nursing home, the nursing staff notes that the patient frequently eats and drinks very little and often coughs after she swallows. Her current medications include fluoxetine, selegiline, carbidopa-levodopa, trazodone, estrogen and sorbitol. Vital signs now are: temperature 36.4°C (97.6°F), axillary; pulse 88/min; respirations 18/min and blood pressure 102/84 mm Hg, supine. Physical examination shows a frail and thin woman. Voice is soft and low. Chest is clear to auscultation. Heart rate is regular without murmurs or gallops. Abdomen is soft and nontender with decreased bowel sounds. Rectal examination is normal. She is generally stiff with low amplitude tremors at rest. Laboratory results show:
Serum
BUN
47 mg/dL
Creatinine
1.9 mg/dL
Na+
161 mEq/L
K+
3.9 mEq/L
Cl-
120 mEq/L
HCO3-
24 mEq/L
When the nursing home calls you with the laboratory results, the office staff informs you that the patient's husband telephoned earlier that afternoon and "fired" you from the case. There are no written notes from the staff and you have not spoken to the husband yourself.

Item 1 of 2

22. The most appropriate next step is to contact which of the following?

A) Husband and give him the names of other physicians who attend patients in the nursing home
[B) Husband and inform him of the laboratory results, their implications and management options]
C) Nursing home and inform them that you are no longer the patient's physician
D) Nursing home and initiate nasogastric rehydration in the facility
E) Nursing home and order the immediate transfer of the patient to the hospital

Item 2 of 2

23. Which of the following complications is most likely to develop if her hypernatremia is corrected too quickly?

A) Disseminated intravascular coagulation
B) Intravascular hemolysis
[C) Pontine myelinolysis]
D) Pulmonary edema
E) Seizures

24. A 68-year-old woman with stage IV ovarian cancer is in the hospital following initial staging and a debulking operation. She refuses chemotherapy and the physician agrees to follow her wishes. This is an example of which ethical principle?

A) Authority
[B) Autonomy]
C) Beneficence
D) Justice
E) Nonmaleficence

25. You assist in the delivery of a neonate born to a couple who have been your patients for the past 7 years. The husband is 50 years old and the wife is 45 years old. The wife had been offered maternal serum triple-marker screening and amniocentesis during the pregnancy because of her advanced age, and she had declined the testing. At the delivery, the neonate has features suggestive of Down syndrome. Apgar score is 9 at 1 minute and 9 at 5 minutes. Initial physical examination demonstrates no cyanosis and no heart murmur. In the delivery room, which of the following is the best initial statement to make to the parents?

A) "Do you remember the tests we discussed early in your pregnancy?"
B) "I am concerned about your infant because she may have Down syndrome."
C) "I am sorry to have to tell you this, but I suspect your baby has Down syndrome."
D) "We are going to have to take the baby to the nursery for some tests."
[E) "Your daughter is doing well, but I am concerned about some of her physical features."]

26. A 26-year-old primigravid woman at 38 weeks' gestation is admitted to the hospital because she is in labor; contractions occur every 3 minutes and last 60 seconds. The patient's prenatal course has been uncomplicated. Labor curve is now normal and fetal heart rate tracing shows good variability with an occasional mild deceleration. Her patient chart indicates that a previous physician thought she might have a platypellic pelvis. She is now 10-cm dilated. The fetus is at +1 station with a mentum-anterior face presentation. Which of the following is the most appropriate management at this time?

A) Deliver the child vaginally after manually rotating the fetus to a mentum-posterior presentation
B) Deliver the child vaginally with the aid of forceps
C) Deliver the child vaginally with the aid of vacuum extraction
[D) Deliver the child vaginally without intervention]
E) Deliver the child via emergency cesarean delivery

27. A full-term, 3402-g (7-lb 8-oz) neonate aspirated meconium at delivery. Apgar score is 3 at 1 minute and 3 at 5 minutes. He is intubated and assisted ventilation is established using a respirator. His condition stabilizes for 30 minutes but the arterial blood gas values suddenly deteriorate. Which of the following is the most appropriate first step?

[A) Auscultate the chest]
B) Check the respirator settings
C) Increase the oxygen flow
D) Recheck the arterial blood gas values
E) Reposition the endotracheal tube

28. A 56-year-old man who is alcohol-dependent is admitted to the hospital because of fever, encephalopathy and increasing abdominal girth. The patient has no family and has been living on the street. An abdominal paracentesis is consistent with spontaneous bacterial peritonitis, which is treated with antibiotic therapy. During the next 10 days, the patient's renal function deteriorates and his serum creatinine concentration increases to 11.3 mg/dL, with a serum urea nitrogen (BUN) concentration of 144 mg/dL. He becomes progressively more obtunded, develops respiratory failure and is intubated. He requires vasopressors to maintain his blood pressure. There is general agreement from the gastrointestinal and renal services that he will not recover and that further treatment, including dialysis, is futile. The patient is thrashing about in bed, moaning and grimacing periodically. Which of the following is the most appropriate next step in management?
A) Do a liver biopsy to identify the cause of the patient's liver failure
[B) Initiate lactulose treatment to reduce his encephalopathy]
C) Initiate analgesic therapy to palliate the patient's apparent discomfort
D) Stop all antibiotic therapy to reduce the chance of further renal toxicity
E) Stop all medical treatment

The following vignette applies to the next 2 items.


A 38-year-old woman is admitted to the hospital for an elective breast-reduction operation. She is in good health and has never been admitted to the hospital or had previous operations. Her only medication is an oral contraceptive pill. Shortly after the administration of general anesthesia she develops muscular rigidity, temperature increases to 40.6°C (105.0°F), pulse increases to 130/min and blood pressure decreases from 110/70 mm Hg to 80/40 mm Hg. Arterial blood gas values show:

PO2
60 mm Hg
PCO2
40 mm Hg
pH
7.12
Total CO2
18 mm Hg
O2 saturation
88%
Item 1 of 2
29. Which of the following is the most appropriate intravenous pharmacotherapy to administer at this time?

[A) Dantrolene]
B) Diazepam
C) Epinephrine
D) Magnesium sulfate
E) Succinylcholine

Item 2 of 2

30. Which of the following is the most likely cause of this reaction?

[A) Abnormal increase in intracellular calcium]
B) Hypothalamic dysfunction
C) Hypovolemia
D) Inability to sweat
E) Inhibition of protein synthesis

The following vignette applies to the next 2 items.


You are asked to see a 63-year-old woman who is in the hospital after undergoing an operation 4 days ago for a displaced tibia-fibula fracture. The operation was uncomplicated. Preoperative chest x-ray film was normal. She has a transurethral urinary catheter, which was placed during the operation 4 days ago. Today she has a temperature of 37.9°C (100.2°F). Laboratory studies disclose mild leukocytosis. Urinalysis shows greater than 100 WBC/hpf and many bacteria. You suspect that she has a urinary tract infection.

Item 1 of 2

31. Which of the following would have been the most appropriate method of preventing a urinary tract infection in this patient?

A) Cleaning the perineum twice daily with povidone-iodine
B) Irrigating the bladder every 8 hours
[C) Removing the catheter immediately post-operation]
D) Treating the patient with prophylactic antibiotics
E) Using only silver-coated urinary catheters

Item 2 of 2

32.Incidence of Bacteruria Related to Catheter Use

Study Randomized Suprapubic Catheter Transurethral Catheter Odds Ratio +95% CI

A No 2/25 21/31 0.04(0.01-0.24)

B Yes 10/48 20/44 0.32(0.11-0.86)

C Yes 2/32 16/34 0.08(0.01-0.41)

D Yes 8/32 5/40 1.87(0.48-8.01)

*CI=confidence interval


A Colleague suggests to you that suprapubic catheters can decrease a patient's risk for bacterial colonization. You identify four studies in the medical literature that compare the incidence of bacteruria in patients with a transurethral catheter versus patients with a suprapubic catheter. The study results are shown. Which of the following is the most important criterion related to the validity of these data?

A) Confidence interval
B) The number of patients with bacteriuria
C) Odds ratio
[D) Sample size]
E) Study design


The following vignette applies to the next 2 items.


A 67-year-old woman with a 2-month history of cough was admitted to the hospital after chest x-ray film and CT scan showed a lung mass with suspected mediastinal extension. Biopsies obtained during bronchoscopy and mediastinoscopy showed poorly differentiated squamous cell carcinoma in the peritracheal lymph nodes. A pulmonary consultant has determined that the tumor is inoperable, and that the only available treatment is palliative radiation therapy. The patient has been informed of the diagnosis by you and others, but when you see her now in your office she seems unclear about what she has been told.

Item 1 of 2

33. Given the patient's uncertainty about the details of her diagnosis, which of the following is the most appropriate next step?

A) Ask the pulmonologist to explain the diagnosis to her
B) Assume that she is depressed and begin antidepressant pharmacotherapy
C) Give her the phone number for the American Cancer Society
[D) Recommend that she bring family members with her to her visit next week]
E) Recommend that she obtain a second opinion

Item 2 of 2

34. The patient returns to the office for follow-up 2 weeks later. She requests copies of her medical records and angrily states, "Doctor, I think you're wrong. I've been praying and I know I don't have cancer." Which of the following is the most appropriate response?

A) "I told you that all of your years of smoking would put you at risk for lung cancer."
[B) "I would be happy to go over all of the reports with you; I appreciate that this is a difficult diagnosis to accept."]
C) "The request for medical records can only come through your attorney."
D) "The sooner you accept this diagnosis, the sooner we can get on with appropriate therapy."
E) "We are having a case conference later today with the pulmonary and radiation therapy specialists; maybe you would like to attend to hear their opinions about your disease."

35. A 45-year-old homeless man is admitted to the hospital because of myalgias and jaundice. He has no known past medical history and denies alcohol or drug abuse. Vital signs are: temperature 37.1°C (98.8°F), pulse 90/min, respirations 18/min and blood pressure 117/75 mm Hg. On examination he appears malnourished. Pupils are round and reactive with scleral icterus. Dentition is poor. Gums are swollen and friable. Neck is supple without adenopathy or bruits. No jugular venous distention is noted. Lungs are clear. Cardiac examination demonstrates a normal S1 and S2 with a 2/6 systolic murmur at the left sternal border. Abdomen is distended, but otherwise normal. Examination of the skin discloses diffuse perifollicular hemorrhages on the legs with purpura and ecchymosis. There is no clubbing or cyanosis of the hands, but splinter hemorrhages of the nail beds are noted. Neurologic examination shows normal cranial nerves. Sensation to touch in the legs is decreased, bilaterally. Laboratory studies show:

Serum
Blood
AST
120 U/L
Hemoglobin
9.2 g/dL
Bilirubin
4.2 mg/dL
WBC
5500/mm3

MCV
82 μm3
Platelets
234,000/mm3
PTT
23 sec
PT
12 sec

INR
1.05
Which of the following is the most likely underlying cause for this patient's symptoms and laboratory findings?
A) Acute viral hepatitis
B) Endocarditis
[C) Vitamin C deficiency]
D) Hemochromatosis
E) Vitamin K toxicity

36. An 8-hour-old neonate, who was born via cesarean delivery at 38 weeks' gestation, is in the neonatal unit. The cesarean delivery was done due to fetal distress. The mother is a healthy 25-year-old woman and this was her first pregnancy. There were no complications during the pregnancy, but bloody amniotic fluid was noted at the time of delivery. The neonate's Apgar score was 8 at 1 minute and 9 at 5 minutes; points were taken off for color only. At 30 minutes of age the infant was noted to have some mild grunting and tachypnea. These symptoms resolved spontaneously during approximately a 40-minute period, but the on-call physician ordered a complete blood count and blood culture for suspected sepsis. Because the symptoms resolved so quickly, no antibiotic therapy was started. You are now satisfied that further pursuit of infectious problems is not indicated. The neonate appears normal, and the mother's initial attempts at breast-feeding seem successful. Laboratory studies on the neonate return and are normal except for a hematocrit of 40% (N=4565). Maternal and infant blood type are both O, Rh-positive. The best course of action at this time is to do which of the following?
A) Obtain hematology consultation
B) Obtain serum iron studies, including total iron-binding capacity
[C) Order an Apt test on the neonate's stool]
D) Order hemoglobin electrophoresis
E) Repeat the hematocrit determination

Reply
#7
Block 3
1. A 60-year-old white woman comes to the office because of a 6- to 12-month history of weakness. She initially noted the weakness in her right hand, and it has now become generalized and is associated with muscle cramps. She is not taking any medications. Past medical history is unremarkable. On physical examination there is marked muscle wasting of the hands and arms bilaterally (left greater than right). There is less muscle wasting in the legs. Sensation to touch remains intact. Muscle fasciculation of the arms and hands is noted and deep tendon reflexes are decreased. Speech and mentation are intact. Which of the following studies is most likely to be abnormal in this patient?

[A) Edrophonium chloride test]
?B) Electromyography
C) Serum protein electrophoresis
D) Urine lead levels
E) Urine mercury levels

2. A 23-year-old primigravid woman comes to the office to begin prenatal care. She and her husband had been having unprotected sexual intercourse for 8 months prior to becoming pregnant. The patient is excited about the pregnancy. She is at 10 weeks' gestation based on the date of her last menstrual period. Medical history is significant for a 6-year history of intermittent left-sided seizures that began following a motor vehicle accident during high school. The patient has taken valproic acid therapy for the past 4 years with good control of the seizures. Vital signs today are temperature 36.7°C (98.0°F), pulse 80/min, respirations 20/min, and blood pressure 110/60 mm Hg. Which of the following is the most appropriate modification to the patient's pharmacotherapy?

A) Discontinue the valproic acid until after pregnancy
B) Switch the valproic acid to diazepam
C) Switch the valproic acid to phenobarbital
D) Switch the valproic acid to phenytoin
[E) No modification is indicated]

3. A 32-year-old secretary comes to the office for a periodic health evaluation and Pap smear. She says, "I've been doing fine except that for the past 2 months my right hand sometimes has been tingling and feels numb, especially at night. I'm always dropping things, too." She takes oral contraceptive pills and thyroid replacement medication for hypothyroidism. She is right-hand dominant. She has no history of acute trauma. On physical examination, she has numbness and tingling in the thumb and first two digits of her right hand after 15 seconds of a wrist flexion test. The remainder of the examination is normal. Which of the following studies is most likely to establish a diagnosis?

A) Cervical spine x-ray films
B) MRI of the brain
[C) Nerve conduction studies]
D) Serum thyroid-stimulating hormone (TSH) concentration
E) X-ray film of the right hand and wrist

4. A 48-year-old Iranian American man comes to the office because of chronic dry cough, weight loss and intermittent temperatures to 38.3°C (101.0°F) for the past 2 months. He has lost 3.5 kg (8 lb) during this time. Today vital signs are: temperature 38.3°C (100.9°F), pulse 90/min, respirations 18/min and blood pressure 120/78 mm Hg. Auscultation of the lungs discloses fine crackles throughout the lung fields. The remainder of the physical examination is normal. Chest x-ray film is shown. Therapy with isoniazid, rifampin, pyrazinamide and ethambutol is begun. Your office staff ask you whether they should be evaluated for exposure to tuberculosis. Which of the following is the most appropriate next step?

A) Do nothing for office staff because the patient is unlikely to be infectious
B) Monitor the staff during the next few weeks for development of cough or fever, and base further treatment on this information
C) Obtain chest x-ray films in 4 weeks of each staff member who came into contact with the patient
[D) Place a 5-TU PPD skin test in 4 weeks on each staff member who came into contact with the patient]
E) Prescribe prophylactic isoniazid therapy to all office staff

5. A 67-year-old man comes to the office for an initial visit. He says his daughter, who is a patient in your practice, "made me come" because of a lesion in his mouth under his tongue. He is a farmer from the South who is visiting his daughter, and he is not concerned about the lesion. He says, "It has been present for a couple of months, and although I can feel it with the tip of my tongue and it is slightly sore, it doesn't give me any trouble." He has always been healthy and takes no medication. He drinks alcohol in small amounts on weekends, and he uses tobacco in a variety of forms. On physical examination, a 2H2-cm, raised, roughened gray lesion on the oral mucosa of the left side of the floor of the mouth is noted extending to the base of the tongue. You decide the lesion is highly suggestive of a malignancy and that a biopsy should be obtained. Based on the patient's history and incidence of lesions in this location, the lesion is most likely which of the following?

A) Adenocarcinoma
B) Basal cell carcinoma
C) Leukoplakia
D) Lymphoma
[E) Squamous cell carcinoma]

6. A 52-year-old African-American man returns to the health center for a follow-up visit 1 week after he was discharged from the hospital after being diagnosed with acute pneumonia. Discharge medications included inhaled bronchodilators, oral antibiotics and a tapering course of oral corticosteroids. The antibiotic and corticosteroid therapies were completed yesterday. He has no previous hospital admissions for respiratory disease. He has a nonproductive, chronic, daily cough. He smoked one to two packs of cigarettes per day since age 13 years, but he has not smoked since admission to the hospital. Vital signs today are: temperature 36.8°C (98.2°F), pulse 92/min, respirations 10/min and blood pressure 120/80 mm Hg. On physical examination he appears well. There are markedly diminished, but clear, breath sounds with no wheezing or rhonchi; there is a prolonged expiratory phase. There is no digital clubbing. Which of the following is the most likeley diagnosis?

A) Asthma
B) Bronchiectasis
[C) Chronic bronchitis]
D) Emphysema
E) Pulmonary fibrosis

The following vignette applies to the next 3 items.


Patient History

Sex: Male

Age: 53 years

Sociodemographic information: African-American high school teacher, married, no tobacco use, history of heavy alcohol use but not for the past 10 years

Medical history: Hypertension; peptic ulcer disease 15 years ago; pneumonia-related hospitalization 15 years ago; no known drug allergies

Family history: Mother died of renal failure due to diabetes and hypertension at age 69 years; father died of heart failure and hypertension; two sisters have hypertension

Current medications/drug information: One aspirin tablet daily

Today's Visit

Height: 185 cm (6 ft 1 in)

Weight: 87 kg (192 lb)

Vital signs:

Temperature 36.8°C (98.2°F) Respirations 16/min

Pulse 88/min, regular Blood pressure 150/95 mm Hg

Physical examination: Blood pressure is equal in both arms, fundi show arteriolar narrowing; remainder of physical examination is normal


Laboratory studies: Electrocardiogram shows normal sinus rhythm and left ventricular hypertrophy


The patient whose chart is shown comes to the office for the first time because he has been transferred to your managed care organization. He had been taking a blood pressure medication but explains that he ran out of it 8 months ago. He did not make an appointment until now because, he says, "I just never got to it." His wife persuaded him to seek care now. He tells you that he requested his previous records be sent to you but they have not arrived.

Item 1 of 3

7. Which of the following is the most appropriate pharmacotherapy for this patient?

A) Clonidine
B) Enalapril
[C) Hydrochlorothiazide]
D) Nifedipine
E) Terazosin

tem 2 of 3

8. The patient returns to the office in 1 week for a follow-up visit. He says he has a cold with cough, nasal congenstion and a scratchy throat. Blood pressure in now 140/85 mm Hg. On physical examination lungs are clear and there is no peripheral edema. Regarding over-the-counter medications for this patient's cold, he should be cautioned to avoid which of the following?

A) Chlorpheniramine
B) Dextromethorphan
C) Guaifenesin
D) Oxymetazoline
[E) Pseudoephedrine]

Item 3 of 3

9. Regarding his blood pressure, which of the following is the most appropriate management?

[A) Add a second antihypertensive medication to the regimen]
B) Add potassium chloride supplements to the medication
C) Change the antihypertensive medication
D) Continue with the current regimen
E) Increase the dose of the prescribed medication
10. A family of two adults and two school-aged children comes to the office because each family member has had intermittent nausea and diarrhea during the past 4 to 6 weeks. They are at increased risk for giardiasis if which of the following is true?

A) A close family friend has AIDS
[B) Their water supply is a shallow well]
C) They frequently consume uncooked vegetables
D) They recently adopted a new pet dog
E) They recently returned from a trip to Mexico

11. A 61-year-old Dutch businessman comes to the office to discuss the results of screening flexible sigmoidoscopy done 1 week ago. He has hypertension and hypercholesterolemia, for which he takes hydrochlorothiazide and pravastatin. His grandmother died of colon cancer, but no other family members have had colon cancer or polyps. During the sigmoidoscopy, the colon was well visualized to 60 cm and a 1-cm polyp was removed. On pathologic examination, the polyp was found to be adenomatous. Which of the following is the most appropriate diagnostic study at this time?

[A) Colonoscopy]
B) Determination of serum α-fetoprotein concentration
C) Determination of serum carcinoembryonic antigen concentration
D) Double-contrast lower gastrointestinal barium study
E) No further study is indicated

12. A 52-year-old woman is brought to the office by her husband for evaluation of increasing tearfulness, restlessness, difficulty sleeping and weight loss for 1 month. During the visit, the patient appears restless and upset. She acknowledges her husband's account of her symptoms, but she does not volunteer any additional information. They have been married for 15 years and have a good relationship. Both the patient and her husband are teachers and they have a healthy 13-year-old son. She was previously healthy. Physical examination today is normal. The most appropriate initial management of her symptoms is to do which of the following?

A) Prescribe haloperidol
B) Prescribe lithium
[C) Prescribe lorazepam]
D) Prescribe sertraline
E) Refer them for couples therapy

13. A 14-year-old white girl is brought to the health center by her mother. The mother is in tears and states, "My daughter has been suspended from school for cursing at a teacher in class. Last night she came home 2 hours after her curfew. When I confronted her she yelled at me and pushed me into the wall. I don't know what is wrong with her." The girl states, "There is nothing wrong with me. I just want to be with my friends and do what I want. I don't feel I need a curfew. I didn't do anything wrong at school. The teacher mouthed off at me. She should have been suspended instead of me." The girl has been your patient since age 10 years when the family moved from another state. She has a history of allergies to mold and dust for which she uses loratadine as needed. She takes no other medications. Her menses began at age 12 years. Vital signs now are: temperature 37.0°C (98.6°F), pulse 90/min, respirations 12/min and blood pressure 106/60 mm Hg. Physical examination is normal. Which of the following is the most appropriate opening question to the patient when you are speaking alone with her?

A) "Are you using drugs or alcohol?"
B) "How do you want your parents to react to your violating their rules?"
[C) "What seems to be bothering you the most?"]
D) "Why aren't you showing your parents any respect?"
E) "Why did you push your mother?"

14. A 7-year-old white boy is brought to the health center by his mother because of stomach pain for the past 3 months. The mother says that the pain typically comes on soon after waking and is not relieved by antacids. She says his symptoms never occur late in the day, but only in the mornings. There has been no vomiting, diarrhea or weight loss. The child has been generally healthy and is up-to-date with vaccinations. He lives with both parents. His height and weight are at the 75th percentile. Physical examination, including developmental assessment, is normal. Specific additional history should be obtained regarding which of the following?

[A) Days of the week in which the patient is most symptomatic]
B) Family history of milk allergy
C) Quantity of food intake
D) Recent travel history
E) The source of water at home

15. A 49-year-old man, who is accompanied by his son, comes to the office for the first time for follow-up of mild depression. He recently moved to the area and he tells you that his previous physician initiated fluoxetine therapy 4 weeks ago. He has a history of stage I malignant melanoma, which was treated with wide, local excision 2 years ago. He mentions that he was adopted as a young child and his family history is unknown. The patient is friendly and engaging, seems to be full of energy and calls you by your first name. He reports that he has been unusually productive at work recently and is able to "get by" on 1 to 2 hours of sleep nightly. His son says, "Dad isn't acting like himself. He talks about weird things and he's become religious in a creepy way." On physical examination the patient is well developed and well nourished. Vital signs are: temperature 37.0°C (98.6°F), pulse 95/min, respirations 14/min and blood pressure 135/90 mm Hg. There is a mild, nonpainful increase in tactile sensory acuity. He says, "My skin is really sensitive." The remainder of the physical examination, including neurologic examination, is normal. Which of the following is the most likely precipitant of this episode?

A) Family conflict
?[B) Fluoxetine]
C) Recurrence of melanoma in his central nervous system
D) Stress
E) Unknown; this cannot be determined

16. A 62-year-old man returns to the office to discuss findings of x-ray films of the lumbosacral spine obtained 2 months ago following an office visit for evaluation of chronic low back pain. Physical examination at that time was normal. He was instructed to take aspirin as needed for pain and to await notification from the office regarding when he should return to discuss the x-ray films. The office staff failed to schedule a return visit for the patient; today's visit was scheduled by the patient. The patient says, "I've been extremely anxious about the potential results of the x-ray film, and I'm angry that nobody from the office called me to schedule a follow-up visit." The radiologist's report accompanying the x-ray film notes the presence of a lytic lesion at the L1 vertebra consistent with metastatic cancer and the recommendation that a CT scan be obtained. In addition to apologizing to the patient for neglecting to contact him to schedule follow-up, which of the following is the most appropriate approach to informing the patient of the x-ray film findings?

A) Inform him of the findings and emphasize that the responsibility to schedule a follow-up appointment was his
[B) Inform him of the findings and emphasize the need for further evaluation]
C) Inform him of the findings and tell him that the radiologist is as much to blame for not informing him of the urgency of his condition
D) Inform him that the findings are nonspecific and that further studies are needed to identify a diagnosis
E) Tell him that the radiologist's report is likely overstated and that additional x-ray films will need to be obtained

17. A 21-year-old woman who is a varsity basketball player comes to the student health center because of left knee pain for the past month. She says the pain has progressively worsened and has not been relieved with ibuprofen. The pain occurs when she descends stairs or is in class for long periods. She says, "My knee pops a lot." Physical examination today shows knee crepitus, and weakness and atrophy of the vastus medialis muscle. Which of the following is the most appropriate treatment at this time?

A) Anterior cruciate ligament reconstruction
B) Arthroscopic meniscectomy
C) Immobilization of the knee
D) Nonweight-bearing on crutches for 2 weeks
[E) Quadriceps-strengthening exercises]

18. A 39-year-old African-American woman is brought to the health center at 7:00 PM by her children because of weakness and numbness in her right leg. She also says, "I have an odd feeling in my left leg." The symptoms started several days ago and gradually have increased. She also experienced low back pain about 10 days ago, which responded until recently to oral morphine and ibuprofen prescribed by her primary care physician, whom she could not reach tonight. She has had no problems with bowel or bladder function. She was treated 4 months ago with mastectomy, chemotherapy and radiation therapy for infiltrating ductal carcinoma of the left breast. Neurologic examination today discloses weakness of all muscle groups in the right leg and thigh, with normal strength in the left leg. There is marked decrease in sensation to pinprick and temperature up to the area of the inguinal ligament on the right, and decreased perception of position and vibration sense in the left lower extremity. The patellar and Achilles reflexes are hyperactive on the right and those on the left are normal. Rectal sphincter tone is normal. Diagnostic studies show metastatic disease to L2 with minimum invasion of the spinal canal. Which of the following is the most appropriate method to control her pain?

A) Chemotherapy
B) Dorsal rhizotomy
?C) Long-acting oral morphine therapy
D) Radiation therapy
[E) Surgical decompression]

19. A 55-year-old man comes to the office because of a 1-month history of right knee pain. He reports no recent trauma to his knee but says that he has recently increased his weekly running distance from 12 miles to 30 miles. On physical examination he has aching knee pain with squatting. The rest of the physical examination is normal. Which of the following factors in the patient's history , if present, would indicate the patellofemoral joint as the source of the pain?

A) Associated thigh pain
B) Buckling of the knee
[C) Increased pain with stair climbing]
D) Locking of the knee
E) Prominent knee swelling

20. A 24-year-old professional gardener comes to the office because of a generalized, maculopapular, itchy rash that developed a few days ago. The itching is keeping him awake at night. Three weeks ago he began treatment for a seizure disorder with 400 mg of phenytoin, daily. Physical examination now shows a generalized maculopapular rash with excoriations. The most appropriate management at this time is discontinuation of the drug and addition of which of the following?

A) Oral hydroxyzine
B) Oral prednisone
[C) Topical betamethasone]
D) Topical emollients
E) Topical hydrocortisone

21. A 58-year-old white man comes to the office because of a mass in his neck for the past 2 months. He says, "My voice is bad and sometimes I can't catch my breath." He is a construction worker and has a long history of alcohol abuse and cigarette smoking. Vital signs are normal. Physical examination shows a large mass in the floor of the mouth, extending to the tonsillar fossa. A 4H5-cm mass is noted in the right anterior triangle of the neck. The greatest immediate risk to the patient is which of the following?

[A) Airway obstruction]
B) Aspiration
C) Bleeding
D) Sepsis
E) Stroke

22. A 15-year-old girl comes to the office because of a 2-day history of an itchy rash on her right ankle. The rash occurred 1 day after she took a school field trip to a local park while wearing a new pair of sandals. She has moderate to severe asthma, for which she takes zafirlukast, salmeterol, and albuterol by metered-dose inhaler as needed. She has been tapering off oral prednisone for the past 2 months. Vital signs are temperature 37.5°C (99.5°F), pulse 86/min, respirations 16/min, and blood pressure 125/80 mm Hg. Auscultation of the lungs discloses mild, diffuse expiratory wheezing. Her right ankle is shown. Which of the following is the most likely diagnosis?

A) Chigger infestation
B) Contact dermatitis
C) Herpes zoster
[D) Poison ivy dermatitis]
E) Scabies

The following vignette applies to the next 2 items.


A 29-year-old truck driver comes to the health center for advice regarding his serum cholesterol concentration, which was determined recently at a shopping mall health fair. He had the test at the request of his wife, who is pregnant with their first child. The test result read "high," with advice to see a physician. The patient has a family history of premature coronary artery disease. Two uncles had myocardial infarctions (MI), one at age 41 years and the other at age 45 years. His father died of an MI at age 47 years. He tells you that his 34-year-old brother takes "some type of cholesterol pill." The patient has no significant medical history. He denies smoking cigarettes but admits to drinking a few beers in the evenings at home. Two to three times per week he meets his friends at a local tavern before driving home. He is currently taking no medications, and his physical examination is normal except for mild obesity. A fasting serum lipid profile was done 2 days ago at the suggestion of the nurse who took his history by phone. Results are available today and show:


Serum
Cholesterol
Total
171 mg/dL
HDL
29 mg/dL
LDL
117 mg/dL
Triglycerides
156 mg/dL

Item 1 of 2

23. To evaluate his risk for cardiac disease, in addition to his serum lipid concentrations it is appropriate to obtain which of the following serum laboratory studies?

A) Angiotensin II concentration
[B) Apolipoprotein B concentration]
C) Endothelin concentration
D) Homocysteine concentration
E) Lipoprotein lipase concentration

Item 2 of 2

24. With respect to his alcohol intake, it is most appropriate to do which of the following?

A) Determine if his uncles and father drank and whether it affected their lipid levels
B) Encourage him to continue drinking in moderation given his lipid analysis
[C) Investigate the risk of his drinking while driving his truck]
D) Recommend that he switch from beer to red wine
E) Recommend that he switch to a low-calorie beer

25. A 50-year-old African-American woman returns to the office for follow-up of diabetes mellitus, which has been treated with diet; extended-release glipizide, 10 mg daily; and metformin, 500 mg twice a day. She says, "I do the best I can adhering to my diet." She tests her blood glucose concentration daily. For the past month her fasting blood glucose concentrations have averaged 170 mg/dL. Hemoglobin A1c 1 week ago was 8.4%. The patient is 167.5 cm (5 ft 6 in) tall and weighs 86 kg (190 lb). Which of the following is the most appropriate change in therapy?

A) Add chlorpropamide
[B) Add insulin]
C) Increase the metformin dosage
D) Stop the glipizide and metformin and start insulin
E) No change is indicated

26. A 36-year-old African-American woman with hypertension comes to the office because of a 3-day history of intermittent, painless, gross hematuria. Symptoms started spontaneously, and she has not had fever, chills, flank pain or dysuria. Her only medication is hydrochlorothiazide. Vital signs are: temperature 37.0°C (98.6°F), pulse 72/min and blood pressure 128/84 mm Hg. Physical examination is normal; there is no costovertebral angle tenderness. Laboratory studies show:

Serum
Urine
Electrolytes
Normal
Protein
3+
Glucose
78 mg/dL
Blood
Positive
BUN
8 mg/dL
Microscopic
RBCs too numerous to count
Blood
No WBCs, bacteria or casts
CBC
Normal
Nitrite
Negative
Partial thromboplastin time
Normal
Urine culture
Pending
Platelet count
268,000/mm3


Prothrombin time
Normal

Which of the following is the most appropriate next step?
[A) Cystoscopy]
B) Determination of protein excretion in a 24-hour urine sample
C) Initiation of antibiotic therapy
D) Renal biopsy
E) Transvaginal ultrasonography of the uterus

The following vignette applies to the next 2 items.


A 15-year-old girl is brought to the office for the first time by her mother, who has been your patient for many years. The mother is concerned because her daughter, who had menarche at age 11 years, has not had a period in the past 6 months. Both mother and daughter have kept a record of the girl's periods. The first three periods were irregularly spaced and not painful. Subsequent periods were regularly spaced and were accompanied by severe cramping on the first day. The cramping subsided 1 year ago. The mother insists that you interview her and her daughter together. She says that her daughter is not sexually active. She tells you that her daughter gets straight A grades and that she is very active in sports. The girl is also a cheerleader and is taking a night course for college credit at the local community college through the school's gifted child program. The mother tells you that her daughter has a good appetite for healthy food and does not eat junk food. She asks you what is causing her daughter's amenorrhea.

Item 1 of 2

27. Which of the following is the most appropriate initial response?

A) "In young women, missing a few periods is not unusual, and we should wait a while longer before becoming concerned."
B) "It is a well-known fact that the most likely cause of amenorrhea in young women is pregnancy. How can you be certain she has not been sexually active?"
[C) "Many things can cause missed periods. After I examine your daughter, I may be able to provide more insight into the problem."]
D) "Over-involvement in some athletic activities can cause menstrual periods to cease."
E) "Your daughter is involved in a lot of activities and may be under stress. That can certainly be a factor in causing cessation of periods."

Item 2 of 2

28. After responding to the mother's question, you ask the mother to wait outside while you speak privately with the girl and do a physical examination. No additional information is obtained. Vital signs are: temperature 37.0°C (98.6°F), pulse 110/min and blood pressure 90/50 mm Hg. Height is 165 cm (5ft 5 in) and weight is 49 kg (105 lb). Physical examination shows some eroding of the enamel on the girl's rear teeth. Sexual maturation is complete. Abdomen is soft and nontender. Which of the following is the most likely diagnosis?

A) Athletic amenorrhea
[B) Bulimia]
C) Hypothyroidism
D) Pregnancy
E) Stress-induced amenorrhea


29. A 39-year-old white homemaker comes to the office because of abnormal vaginal bleeding. You provide routine medical care for her and her family. She says her menstrual periods always have been regular and that her last normal regular period was about 10 weeks ago. She has had spotting for 1 week. She tells you that she and her husband use "withdrawal" as their birth control method. Urine pregnancy test is positive. Vital signs now are: temperature 37.4°C (99.3°F), pulse 88/min and blood pressure 110/72 mm Hg. Pelvic examination discloses a small amount of blood in the vagina. Cervix is closed. Uterus is soft, nontender and anteverted. Adnexae are nontender. Additional history reveals that she had a first trimester elective abortion at age 20 years and she has genital herpes that recurs about twice each year. Her husband is age 40 years and has Crohn disease. Her children are ages 7 and 9 years. The 7-year-old was born by cesarean delivery at 32 weeks' gestation after placental abruption. This couple's greatest risk factor for miscarriage is which of the following?

A) History of abruptio placenta
B) History of genital herpes
C) History of premature delivery
[D) Maternal age]
E) Paternal history of auto-immune disease

30. A 30-year-old woman who is 28 weeks pregnant with her first child comes to the office for a routine prenatal visit. She has been a strict vegetarian for many years. At each prenatal visit she has insisted that she is eating well. However, she has gained only 0.9 kg (2 lb) since her initial prepregnancy weight of 54 kg (120 lb); she is 165 cm (5 ft 5 in) tall. She says that she has been taking her prenatal vitamins and iron as instructed. She has a regular exercise routine but has decreased this activity significantly during the past 4 weeks because, she says, "I've been too tired." Pulse is 90/min and blood pressure is 110/60 mm Hg. Physical examination shows a reddened, malar rash. Laboratory studies at this visit show:
Hematocrit 25%
Hemoglobin 7.5 g/dL
WBC 3500/mm3
MCV 105 μm3
Platelet count 89,000/mm3
Which of the following is most appropriate to prescribe orally at this time?
A) Corticosteroid therapy
[B) Vitamin Bl2 (cyanocobalamin) supplementation]
C) Folate supplementation
D) Iron supplementation
E) Vitamin E (tocopherol) supplementation

31. A 36-year-old African-American woman with hypertension comes to the office because of a 3-day history of dysuria, hematuria, and urinary frequency and urgency. Symptoms started the day after she returned from a weekend visit with her boyfriend in another city. She has no prior history of urinary tract infections and she denies fever, chills, flank pain, nausea or vomiting. Her only medication is hydrochlorothiazide. Vital signs at today's visit are: temperature 37.6°C (99.6°F), pulse 90/min and blood pressure 126/84 mm Hg. Physical examination is normal. Laboratory studies show:

Urine
Blood Positive
WBC 15-20/hpf
RBC 15-20/hpf
Bacteria Moderate
Casts Negative
Nitrate Positive
Protein 1+
Which of the following is the most appropriate next step?

A) Increase oral intake of fluids and order a urine culture
B) Order cystoscopy
C) Order intravenous urography
D) Repeat the urinalysis in 3 to 4 days
[E) Start antibiotic therapy

32. A 54-year-old white man comes to the office because of increasingly severe nosebleeds for the past 10 days. He says initially his nosebleeds resolved spontaneously, but over the past week they have become more frequent and have had a heavier flow. He says his nose has been bleeding almost constantly for the past 48 hours. There is no history of trauma or previous nosebleeds. His only medication is quinidine, which was started 6 weeks ago for restless legs. Medical history is otherwise unremarkable and he takes no other medications. Vital signs are: temperature 36.5°C (97.7°F), pulse 75/min, respirations 18/min, and blood pressure 132/85 mm Hg. His nose is packed with tissue paper and he has blood stains on his shirt. Examination of the nose shows no active bleeding or nasal lesions, but there is dried blood on his nares. Petechiae are noted inside the oral cavity. Examination of the skin shows scattered ecchymoses over the lower extremities. The remainder of the physical examination is noncontributory. Results of laboratory studies show:

Serum Blood
ALT 32 U/L Hematocrit 37%
AST 43 U/L Hemoglobin 11.7 g/dL
WBC 6500/mm3
PTT 22 seconds
INR 1.0
Platelet count 32,500/mm3

Which of the following is the most appropriate initial management?
A) Bone marrow biopsy
B) Determination of bleeding time
[C) Discontinuation of quinidine]
D) Platelet transfusion
E) Vasopressin therapy

33. At an annual company examination, a 40-year-old man has an elevated mean corpuscular volume. Hemoglobin concentration is normal. This finding is most suggestive of which of the following?

[A) Alcohol abuse]
B) Hemosiderosis
C) Hepatitis B infection
D) Schistosomiasis
E) Vitamin B6 (pyridoxine) deficiency

34. A 15-year-old white girl is brought to the health center by her mother for a periodic health evaluation and refill of tetracycline for acne. Upon entering the examination room, the patient's mother asks to speak alone with you. In private, she says, "My daughter has been like a different person for the past 4 months. She used to be on the honor roll and is now failing school. She received detention six times for truancy and for talking back to teachers. She argues with her brother and me over everything. She quit the tennis team and doesn't see any of her friends." The mother can think of no explanation for these changes. When you interview the patient alone, she becomes angry and says, "Mom should mind her own business and stop talking about me behind my back." She denies use of drugs or alcohol. She tells you, "I don't want to go out of the house because my acne makes me feel ugly. Plus, I've put on ten pounds." Today she weighs 54 kg (120 lb); she is 165 cm (5 ft 5 in) tall. Which of the following is the most appropriate advice to the mother?

[A) Her daughter is just going through a normal adolescent phase]
B) Her daughter would benefit from a diet program
C) Her daughter would benefit from a trial of antidepressant medication
D) She should use a behavior-modification program to control her daughter
E) No advice is indicated because it would violate patient confidentiality

35. A 17-year-old African-American girl comes to the health center because she wants contraception. She has been sexually active for the past 2 years. She has never taken oral contraceptive therapy. She says that her boyfriends use condoms some of the time. Past medical history is unremarkable. She takes no medication and has no allergies. She does not drink alcoholic beverages. She smokes one-half pack of cigarettes per day. Her last menstrual period, which was normal, was 3 weeks ago. She has had no vaginal symptoms. The patient is 165 cm (5 ft 5 in) tall and weighs 62 kg (137 lb). Pulse is 62/min and blood pressure is 106/64 mm Hg. Physical examination, including pelvic examination, is normal. Which of the following is the most appropriate recommendation for contraception in this patient?

[A) Combination oral contraceptive therapy]
B) Diaphragm
C) Intrauterine device (IUD)
D) Levonorgestrel
E) Spermicidal jelly

36. A 20-year-old white woman comes to the office because of increasing fatigue for the past 3 weeks. She is 24 weeks pregnant with her first pregnancy. She says she has noted increased difficulty maintaining her usual pace in the local factory where she works packing boxes. Prior to pregnancy, she had no health problems and was considered an above-average worker. Other than the fatigue and an occasional backache, she has had an uneventful pregnancy. Results of initial prenatal laboratory studies were normal, and she has attended all of her prenatal office appointments. Complete blood count is obtained at this visit and a peripheral blood smear, also obtained during this visit, is shown. At this time, the most likely explanation for the patient's fatigue is that she has which of the following?

[A) Iron-deficiency anemia]
B) Vitamin B12 (cyanocobalamin) deficiency
C) Folate deficiency
D) Leukemia
E) Physiologic anemia of pregnancy
Reply
#8
Block 4
================================================================================
1. A 75-year-old retired landscape architect is brought to the emergency department after the police found him wandering around the city. He was unable to tell them who he was or where he lived. He has a long history of poorly controlled hypertension. He had two strokes several months ago from which he partially recovered. He can walk but his left arm remains weak. Pertinent family history indicates that his father, a successful businessman, committed suicide at age 72 years. Which of the following is the most likely diagnosis?

A) Bipolar disorder, manic episode
B) Major depressive disorder
[C) Vascular dementia]
D) Dementia, Alzheimer type
E) Schizophrenia

2. A 37-year-old woman comes to the emergency department because of a 2-day history of weakness in her left foot which has caused her to accidentally stub the toes of her left foot several times. She reports no other symptoms and is otherwise generally healthy. Medical history is unremarkable except for an episode of blurred vision 1 year ago that resolved on its own. She takes no medications and does not smoke cigarettes or drink alcoholic beverages. Physical examination shows a left footdrop but is otherwise noncontributory. Neurologic examination shows hyperreflexia of the deep tendon reflexes in all extremities. CT scan of the head shows no abnormalities. Lumbar puncture is done. Examination of this patient's cerebrospinal fluid is most likely to show which of the following?

A) Elevated glucose
[B) Elevated IgG]
C) Elevated neutrophil count
D) Elevated protein
E) Positive VDRL

3. A 55-year-old African-American woman is transported to the emergency department by paramedics. She was sitting at the dinner table, and suddenly said to her husband, "I have a terrible headache." She then dropped her fork and slumped in her chair. The husband went to her aid, and she said, "I can't move my right side." He then called emergency medical services. On arrival the patient is alert and oriented. She is holding her head and is in obvious distress. Vital signs are: pulse 101/min, respirations 28/min and blood pressure 190/118z mm Hg. The patient's husband tells you that she has been treated for hypertension for the past 10 years and that she has a 25-year history of simple migraines that occur every 3 to 4 months. On physical examination the patient has an obvious right hemiparesis with an associated hemisensory deficit and hyperreflexia. Blood is drawn for laboratory studies, electrocardiography is done and CT scan of the head is ordered. Her husband asks, "What do you think happened, Doctor?" Based on her clinical presentation and past medical history, which of the following is the most likely cause?

A) Cerebellar hemorrhage
B) Complicated migraine
C) Embolic cerebral infarct
[D) Intracerebral hemorrhage]
E) Subarachnoid hemorrhage

4. A 36-year-old firefighter is trapped in a burning building when his air supply runs out. When the rescuers find him 15 minutes later, he is breathing spontaneously and has a pulse, but he is unresponsive. There is no evidence of acute traumatic injury. He is resuscitated with 100% oxygen via a nonrebreathing face mask. He gradually becomes conscious, and by the time he reaches the emergency department, he is awake, alert and complains only of a headache. On arrival in the emergency department arterial blood gas values while breathing 100% oxygen show:

PO2
493 mm Hg
PCO2
29 mm Hg
pH
7.53
Carboxyhemoglobin
30%
Which of the following is the most appropriate management at this time?

A) Admit the patient for careful monitoring of his airway
B) Begin phenytoin therapy by intravenous loading
[C) Have the patient rebreathe 100% oxygen at 10 L/min]
D) Intubate the patient in order to allow controlled mechanical ventilation
E) Measure the concentration of carbon monoxide in his blood

5. A 23-year-old rugby player is brought to the emergency department by his teammates after sustaining blunt trauma to the left infra-orbital area during a game. The patient complains of significant periorbital pain and says, "I'm seeing flashing lights." Physical examination discloses periorbital swelling. Pupils are equal and reactive. Visual acuity is 20/20 in the right eye and 20/40 in the left eye. Which of the following is the most appropriate step?

A) Discharge and follow-up with an ophthalmologist within 24 hours
B) Discharge with a nonsteroidal anti-inflammatory drug (NSAID) and an eye patch
C) Discharge with corticosteroid eyedrops and a nonsteroidal anti-inflammatory drug (NSAID)
D) Discharge with pain medication and application of ice packs
[E) Immediate consultation with an ophthalmologist]

6. A 19-year-old white college student is brought to the emergency department by her two roommates because of shortness of breath. The roommates tell you that her symptoms came on suddenly about one hour after they picked her up at the airport where she had arrived after a 6-hour flight from visiting her parents. The patient has rapid and shallow breathing, and with difficulty she tells you, "I can't get my breath and I'm having pains in my chest. My face is numb. I think I'm dying. Do something. Do something!" She says she has never had an experience like this before. She always has been healthy and she takes no medications except for combination oral contraceptive therapy. Physical examination is normal except for tachypnea and tachycardia. Electrocardiogram shows sinus tachycardia but is otherwise normal. Arterial blood gas values while breathing room air show:

PO2
99 mm Hg
PCO2
30 mm Hg
pH
7.44

Which of the following is the most appropriate long-term pharmacotherapy?

A) Bupropion
B) Gabapentin
C) Lorazepam
D) Metoprolol
[E) Paroxetine]

The following vignette applies to the next 2 items.

A 58-year-old white man comes to the emergency department because of chest heaviness for the past 45 minutes associated with shortness of breath and a sense of doom. He has not seen a physician since he was 18 years old. He smokes approximately two packs of cigarettes per day. He takes no prescription medications. He has had increasing heartburn recently and he has been taking eight to twelve antacid tablets daily during the past 3 weeks. He is mildly nauseated and diaphoretic. Vital signs are: temperature 37.2°C (99.0°F), pulse 98/min, respirations 20/min and blood pressure 160/96 mm Hg. Electrocardiogram shows that ST-T segment changes are indeterminate. You are aware of a new blood test, CQ, that can diagnose an acute myocardial infarction (MI) more quickly than the creatine kinase isoenzymes. The receiver operating characteristic (ROC) curve for CQ is shown in the exhibit for four cut-off points. You believe the likelihood of an acute MI is high in this patient and you want to minimize the chance of a false negative.

Item 1 of 2

7. Which of the following is the most appropriate cut-off point on the ROC curve?

A) A
[B) B]?
C) C
D) D
E) It cannot be determined with the data provided

Item 2 of 2

8.

The cut-off point at which the test performance for CQ is most accurate in the detection of an acute MI is which of the following?

A) A
B) B
C) C
[D) D]
E) It cannot be determined from an ROC curve

End of Set

9. A 67-year-old man is brought to the emergency department in ventricular fibrillation. His rectal temperature is 26.7°C (80.0°F). Among the following criteria, the patient should be declared dead when defibrillation fails after which of the following?

A) Rapid endotracheal intubation and administration of 100% oxygen
[B) Restoration of a normal core temperature]
C) Restoration of a normal oral temperature
D) 1 hour of core rewarming
E) 20 minutes of warming, externally

10. A 19-year-old white woman is brought to the emergency department by her mother because of intractable emesis. She has a past history of bulimia and according to her mother has been under the care of a psychiatrist. She apparently was well until 1 day ago when she developed emesis and an inability to tolerate liquid and solid foods. Although she takes no medications other than oral contraceptives, she admits to occasional use of both laxatives and ipecac. She denies the use of alcohol or of illicit drugs and says she has experienced no previous symptoms of chest pain, heartburn, hematemesis or fever. Physical examination shows a well-nourished woman with normal vital signs. No lesions are evident on inspection of the skin, but turgor is poor. The thyroid is flat, nontender and without masses. On auscultation of the lungs, moist rales are present at the bases bilaterally. The point of maximal impulse of the heart on the chest wall is 7 cm from the lower left sternal margin in the sixth intercostal space. There is no evidence of an S4 although an S3 is heard. A soft systolic murmur is heard at the apex without a diastolic component. Jugulovenous distention is present 3 cm above the suprasternal notch at 45 degrees' elevation of the chest. The abdomen is soft and a tender liver edge extends 3 cm below the right costal margin. Pitting edema is present in both legs to the mid-calf bilaterally. Laboratory studies show a serum creatinine concentration of 4.2 mg/dL and serum urea nitrogen (BUN) of 88 mg/dL. Urinalysis shows renal tubular epithelial cell casts. Chest x-ray film shows cardiomegaly, central hilar vascular congestion and cephalization of blood flow. Which of the following is the most likely explanation for cardiac decompensation and renal failure in this patient?

[A) Bulimic cardiomyopathy]
B) Cocaine intoxication
C) Hypothyroidism
D) Myocardial ischemia
E) Myocarditis

11. A 73-year-old recently widowed African-American woman comes to the emergency department because of abdominal pain, nausea and constipation for the past 3 days. She says the pain has been intermittent and she has had severe nausea and left-sided stomach cramps. She alternates between being constipated and having diarrhea but she has not had a bowel movement in the past 36 hours. She admits to smoking one pack of cigarettes per day, and she has diabetes mellitus that is poorly controlled with diet and glyburide. Vital signs now are: temperature 37.8°C (100.8°F), pulse 100/min, respirations 28/min and blood pressure 180/90 mm Hg. Abdomen is distended and tender; rectal examination is positive for occult blood. X-ray film is shown. Which of the following is the most appropriate management at this time?

A) Discharge home with clear fluids
B) Exploratory laparotomy
[C) Nasogastric tube]
D) Oral antibiotics
E) Rectal tube

12. A 10-day-old female neonate with Down syndrome is brought to the emergency department by her mother because she has been vomiting for the past 2 days. The vomitus is bile-stained. She was initially breast-fed, but she has been drinking cow milk-based formula for the past week. Physical examination shows slight fullness in the left upper quadrant without obvious tenderness. Test of the stool for occult blood is negative. Which of the following is the most likely diagnosis?

A) Allergy to cow milk protein
[B) Duodenal obstruction]
C) Hypertrophic pyloric stenosis
D) Peptic ulcer with pyloric obstruction
E) Small-bowel volvulus

The following vignette applies to the next 2 items.


A 49-year-old homeless white man comes to the emergency department and says, "I began vomiting 2 to 3 hours ago, and then started to throw up blood." He reports vomiting "about half a cup" of red blood. He had epigastric discomfort after several episodes of emesis, but no preceding abdominal pain. The patient says that he drinks about a half pint of bourbon per day, and he does not use aspirin or nonsteroidal anti-inflammatory drugs (NSAIDs). He has no previous history of similar symptoms. Vital signs now are: temperature 37.0°C (98.6°F), pulse 105/min and blood pressure 150/77 mm Hg. On physical examination he is alert, oriented and disheveled, and he has the odor of alcohol on his breath. No scleral icterus is present. Abdomen is soft, with mild epigastric tenderness. Liver edge is palpated 2 cm below the right costal margin and is nontender. Bowel sounds are present. Stool is negative for occult blood. An intravenous line is started. Endoscopy confirms a tear of the gastroesophageal junction. Laboratory studies show:

Serum
Amylase
135 U/L
BUN
10 mg/dL
Creatinine
0.7 mg/dL
Na+
137 mEq/L
K+
3.3 mEq/L
Cl-
97 mEq/L
HCO-3
22 mEq/L
Blood
Hematocrit
37%
Hemoglobin
12 g/dL
WBC
12,100/mm3
Platelet count
317,000/mm3

Item 1 of 2

13. The patient says, "What are you going to do, Doc?"

Which of the following is the most appropriate next step?

A) Cimetidine, intravenously
[B) Observation and supportive care]
C) Octreotide, intravenously
D) Sclerotherapy
E) Selective arterial vasopressin

Item 2 of 2

14. After 1 hour the patient's condition has stabilized. Despite your urging him to stay for further evaluation, he insists on leaving.


At this time which of the following is most appropriate to tell the patient?

A) An elective operation should be scheduled
B) He is at immediate risk for major gastrointestinal hemorrhage
C) His symptoms might indicate severe alcoholic liver disease
D) Omeprazole would help prevent further episodes
[E) The risk for rebleeding from this episode is relatively small]


The following vignette applies to the next 2 items.


A 17-year-old African-American boy is brought to the emergency department by his mother and two of his friends at 2:00 AM on Sunday morning. His friends had taken him home from a party after he began to act strangely. They were aware that the patient had recently been treated for marijuana abuse, and they believe that he has significantly cut back on his drug use. His friends began to worry when he insisted that several peers at the party were talking about him behind his back and were plotting to harm him. On the way to the hospital, he was adamant that the same individuals were following them and that they were all in great danger. Now, the patient is unable to walk a straight line, and his speech is slurred. Vital signs are: temperature 38.1°C (100.5°F), pulse 130/min, respirations 20/min, and blood pressure 150/105 mm Hg. Physical examination discloses vertical and horizontal nystagmus. The remainder of the physical examination and neurologic examination is normal.

Item 1 of 2

15. The most likely cause of the patient's current symptoms is intoxication with which of the following?

A) Alcohol
B) Cannabis
C) Heroin

D) Nitrous oxide
[E) Phencyclidine]

Item 2 of 2

16. Which of the following is the most appropriate pharmacotherapy?

A) Diphenhydramine
B) Fluoxetine
[C) Haloperidol]
D) Hydroxyzine
E) Phentolamine

The following vignette applies to the next 2 items.

A 4-year-old girl is brought to the emergency department by her mother because of severe wrist pain. The girl was playing with her friends in her backyard and fell, breaking the fall with her outstretched hand. On physical examination there is slight swelling over the dorsal aspect of the wrist. X-ray films of the wrist are shown.

Item 1 of 2

17. The findings are most consistent with which of the following?

A) Carpal navicular fracture
[B) Cortical fracture of the radius]
C) Fracture of the distal radius and ulnar growth plates
D) Fracture of the distal radius growth plate
E) Perilunate dislocation of the wrist

Item 2 of 2

18. Which of the following is the most appropriate management?

A) Apply an elastic bandage and apply ice packs to the wrist
B) Do closed reduction of the fracture
C) Do closed reduction of the fracture and report the case to child protective services
[D) Immobilize the forearm and hand in situ in a cast]
E) Prepare for open reduction and internal fixation


The following vignette applies to the next 3 items.

A 10-year-old boy is brought to the emergency department because he developed hives and shortness of breath 10 minutes after being stung by an insect. His father tells you that he had a similar episode of dyspnea and urticaria 2 years ago. Physical examination now shows a frightened child who appears out of breath, has generalized urticaria and asks for help in a hoarse voice. Vital signs are: temperature 37.0°C (98.6°F), pulse 120/min, respirations 36/min and blood pressure 70/40 mm Hg.

Item 1 of 3

19. Which of the following is the most important first step in managing this patient?

A) Administer diphenhydramine, orally
[B) Administer epinephrine, subcutaneously]
C) Administer oxygen via face mask
D) Establish intravenous access
E) Obtain arterial blood gas values

Item 2 of 3

20. After 10 minutes there is no change in his condition.


At this time, the most appropriate next step is to administer which of the following?

A) Diphenhydramine, orally
B) Dopamine, intravenously
[C) Epinephrine, subcutaneously]
D) Prednisone, orally
E) Ranitidine, orally

Item 3 of 3

21. The patient improves with treatment. In 30 minutes his urticaria, dyspnea and hoarseness are resolved and his vital signs are normal. As you prepare to discharge the patient you recommend that he be evaluated by an allergist.


The patient's mother asks you what should be done until he can be seen by the allergist. Which of the following is the most appropriate recommendation?

A) Avoid all outdoor activity
B) Take diphenhydramine, orally, every 4 hours while awake
C) Take oral prednisone once daily
D) Use an albuterol metered-dose inhaler after any insect sting
[E) Use an epinephrine autoinjector if he is stung again]

22. A 3-year-old girl is brought to the emergency department by her father because of vomiting. He reports that her medical history is unremarkable except for a viral infection 1 month ago, during which she had a mild fever and was irritable for 2 days. She recovered quickly and was well until 3 days ago, when she seemed more thirsty than usual and did not eat as much solid food as she had before. She began vomiting last night and was lethargic today. She is afebrile, pulse is 180/min and respirations are 40/min and deep. On physical examination she is lethargic but responds to touch. Which of the following abnormalities is most likely on further physical examination?

[A) Acetone-smelling breath]
B) Hepatomegaly
C) Multiple bruises of various stages of healing
D) Nuchal rigidity
E) An olive-sized mass in the right upper abdominal quadrant

23. A 71-year-old retired oil refinery worker comes to the emergency department at 2:00 AM because of inability to urinate for the past 6 hours. He says he is having abdominal discomfort and that he has had a decreased urinary stream and urinary dribbling for the past 4 months. Vital signs are: temperature 36.5°C (97.7°F), pulse 103/min and blood pressure 140/90 mm Hg. His lower abdomen is mildly tender and the urinary bladder can be percussed at 2 cm below the umbilicus. Rectal examination shows an enlarged, firm, smooth prostate. Neurologic examination is normal. Which of the following is the most appropriate initial management?

A) Admit the patient to the short-stay unit for observation
B) Do a suprapubic cystostomy and drain the bladder
[C) Insert an indwelling urinary catheter]
D) Order pelvic ultrasonography
E) Order retrograde cystourethrography

24. A 15-year-old girl is brought to the emergency department by her sister. The patient is 36 weeks pregnant and is very upset. She says, "I don't feel the baby move like I used to. Something's wrong!" She has had no prenatal care. A fetal nonstress test is obtained and is nonreactive. Which of the following is the most appropriate first step?

[A) Assess biophysical profile]
B) Determine her hemoglobin concentration
C) Determine her serum glucose concentration
D) Induce labor
E) Order amniocentesis to determine fetal maturity

The following vignette applies to the next 2 items.

A 56-year-old white executive is admitted to the hospital from the emergency department following a severe nosebleed. One month ago he had a brief viral illness after being exposed to an exanthem eruption of one of his grandchildren. At that time the patient was also referred to a urologist because of fatigue, low back pain, and urinary frequency. He was diagnosed with prostatitis, for which he has been taking sulfamethoxazole-trimethoprim for the past 12 days. He does not take any other medications. On arrival in the emergency department vital signs were: temperature 36.8°C (98.2°F), pulse 100/min, respirations 16/min, and blood pressure 120/66 mm Hg. The patient appeared pale with scattered areas of bruising on his limbs and body and a few petechiae. No lymphadenopathy or organomegaly was found. Results of laboratory studies obtained in the emergency department are shown:

Blood
Urine
Hematocrit 21%
WBC 0/hpf
Hemoglobin 5.6 g/dL
RBC 10-20/hpf
WBC 2000/mm3
MCV 102 μm3
Partial thromboplastin time 26 sec
Platelet count 20,000/mm3
Prothrombin time 12.8 sec
INR 1.3

Bone marrow biopsy shows marked hypocellularity.
Item 1 of 2

25. Which of the following is the most appropriate management?

A) Administer granulocyte colony-stimulating factor
B) Administer high-dose short-term corticosteroids
C) Administer parenteral broad-spectrum antibiotics
D) Begin transfusion with whole blood
[E) Discontinue sulfamethoxazole-trimethoprim]

Item 2 of 2

26. Supportive measures are provided for the patient.

Which of the following is the most appropriate treatment recommendation for this patient at this time?

A) Chemotherapy
[B) Follow-up evaluation in 1 week]
C) Glucocorticoid therapy
D) Hematopoietic growth factor therapy
E) Stem cell transplant

27. An 87-year-old woman is brought to the emergency department by ambulance. Her friend found her lying in bed in her home about one-half hour ago. She had been incontinent of urine and had also vomited. The patient has a history of degenerative joint disease, hypertension and chronic obstructive pulmonary disease. The paramedics brought in her medications, which include felodipine, naproxen, albuterol inhaler, ipratropium inhaler, prednisone, theophylline and ciprofloxacin. On questioning the woman she says she has a headache and nausea, but she is not able to give a more coherent history. She appears restless, tremulous and agitated. Vital signs are: temperature 37.0°C (98.6°F), pulse 120/min, respirations 26/min and blood pressure 110/65 mm Hg. Physical examination is normal except for mild expiratory wheezing. Chest x-ray film is normal. Which of the following is the most likely cause of her symptoms?

A) Exacerbation of chronic obstructive pulmonary disease
B) Gastroenteritis
C) Migraine
D) Stroke
[E) Theophylline toxicity]

28. A 53-year-old white man is brought to the emergency department by emergency medical services after he crashed his car into a tree. He was not wearing a seatbelt. Upon arrival in the emergency department the patient is clearly drunk but he is cooperative during the examination. Vital signs are: temperature 37.0°C (98.6°F), pulse 110/min, respirations 18/min and blood pressure 110/75 mm Hg. Physical examination shows generalized tenderness over the lower abdomen and pelvis. Neurologic examination is normal. X-ray films of the cervical spine, chest and pelvis are normal, as is CT scan of the head. On reexamination 3 hours later, no urinary output has been recorded. The patient is unable to produce a urine sample. He has received 1400 mL of lactated Ringer solution since the accident. Foley catheter is placed and yields 5 mL of bloody urine. X-ray film obtained after placement of the Foley catheter is shown. Which of the following is the most appropriate next step?

A) Foley catheter drainage for 10 days
B) Observation only
C) Percutaneous nephrostomy
[D) Suprapubic catheter drainage]
E) Surgical repair

29. A 17-year-old white girl is brought to the emergency department after she was struck by a car while riding her bicycle. She was wearing a helmet. She is awake, alert, and oriented. Vital signs are temperature 37.0°C (98.6°F), pulse 100/min, respirations 18/min, and blood pressure 107/60 mm Hg. Pulse oximetry shows an oxygen saturation of 96% while breathing room air. Physical examination shows no cervical spine tenderness. Breath sounds are clear. Abdominal, pelvic, and neurologic examinations are normal. Screening x-rays of the lateral cervical spine and pelvis are normal. Chest x-ray is shown. Which of the following is the most likely diagnosis?

A) Aortic rupture
B) Flail chest
C) Hemothorax
D) Perforated viscus
[E) Pulmonary contusion]

The following vignette applies to the next 3 items.

You are notified that your patient, a 26-year-old pregnant woman, has been brought by ambulance to the emergency department after she was in an automobile accident. The vehicle in which she was a passenger was broad-sided by another car. She was in the front passenger seat and was wearing a lap/shoulder belt. You have known the patient for 10 years. She is at 34 weeks' gestation with her second pregnancy; she has one child. The nurses attach an external fetal monitor immediately upon the patient's arrival. When you arrive, the patient's vital signs are: pulse 110/min, respirations 18/min and blood pressure 120/80 mm Hg. The fetal heart rate is 150/min with occasional accelerations to 160/min and no decelerations. The monitor shows uterine contractions about every 7 minutes. The patient states that her only discomfort is from the contractions. She says, "They feel like the hard ones from the end of labor with my other baby." On physical examination, the abdomen is very tender to palpation. On speculum examination, there is a small amount of bright red blood oozing from the cervix, which is long and closed. The patient asks you how long she will have to stay in the hospital.

Item 1 of 3

30. Which of the following is the most appropriate response at this time?

[A) "We need to monitor both you and the baby and do some additional tests before I can answer your question."]
B) "You are in preterm labor from the accident. We will try to stop the contractions with medication, and you can go home later today."
C) "You seem a little shaken up, but the baby is fine. I want you to go home but remain in bed the rest of the day."
D) "You seem a little shaken up. Even though the baby seems fine, I would like to keep you overnight for observation."
E) "You will need to stay here until the baby is delivered."

Item 2 of 3

31. While you are talking with the patient, she has a severe contraction that lasts for 5 minutes. Fetal heart tones decrease to 60/min. Which of the following is the most appropriate action at this time?

A) Determine fetal scalp pH
B) Give the mother oxygen by face mask and magnesium sulfate by slow intravenous push
[C) Prepare for immediate cesarean delivery]
D) Place an internal fetal scalp electrode
E) Rupture the membranes artificially for vaginal delivery

Item 3 of 3

32. The appropriate action is undertaken. The patient asks you if she would have been better off if she had not been wearing a seatbelt. Which of the following is the most appropriate answer?

A) "If your car has an air bag, you should not use the seat belt when you are pregnant. But if your car does not have an air bag, we still recommend you use the seat belt."
B) "We prefer that pregnant women not wear seat belts because, in case of accidents, the belt can cause more harm than it prevents."
C) "Pregnant women should use seat belts until about 28 weeks' gestation. After that, the abdomen is so large that the belt can cause the kind of problem you experienced."
[D) "The problem you had was caused from the forces of the accident. You might have been injured more seriously without a seat belt."]
E) "Your problem was that you were sitting in the front passenger seat, which is the most dangerous seat in the car. If you had sat in a different seat, this would not have happened."


33. A 47-year-old man is brought to the emergency department because of the sudden onset of chest pain. On cardiac examination, which of the following physical findings is suggestive of ischemia?

[A) Early diastolic murmur at the base]
B) A late systolic murmur at the apex
C) Mid-systolic click
D) Pericardial knock
E) Pulsus paradoxus

34. A 23-year-old woman comes to the emergency department because of fever. She was diagnosed with acute lymphoblastic leukemia 2 weeks ago, and chemotherapy was initiated 3 days later. She has received all of the chemotherapy as an outpatient. Several hours prior to admission she developed a single episode of shaking chills, which spontaneously resolved. Vital signs are temperature 38.4°C (101.1°F), pulse 108/min, respirations 16/min, and blood pressure 120/80 mm Hg. She is in mild distress. Partial alopecia is noted, and a Hickman catheter is in place. Results of stat complete blood count are obtained and shown:

Blood
Hemoglobin
8.5 g/dL
WBC
950/mm3
Platelet count
80,000/mm3
Urinalysis shows no abnormalities. Two sets of blood cultures are obtained. Which of the following is the most appropriate next step?

A) Administer a single dose of antibiotics and discharge with oral antibiotics
[B) Admit her to the hospital]
C) Discharge her on G-CSF (filgrastim)
D) Monitor vital signs in the emergency department
E) Remove the Hickman catheter and discharge with oral antibiotics

The following vignette applies to the next 2 items.

An 81-year-old Chinese-American woman is brought to the emergency department by her husband because of back pain. She says, "My back hurts and the pain is getting worse." The patient tells you that 2 days ago, an epidural block was done by a staff anesthesiologist because of a chronic, painful left L5 radiculopathy. She obtained temporary relief immediately after the procedure, but about 24 hours ago she began having midline low back pain without radiation that has increased in severity. She was unsuccessful today in contacting the orthopedist who arranged the procedure. Vital signs now are: temperature 38.5°C (101.3°F), pulse 101/min and blood pressure 140/85 mm Hg. On physical examination there is tenderness over the L3-5 area in the midline, which is slightly swollen. Straight leg-raising test is negative bilaterally. Anal sphincter tone is normal. There is decreased sensation over the left lateral calf, an absent left ankle reflex and moderate weakness of left great toe extension. The remainder of the neurologic examination of the leg is normal.

Item 1 of 2

35. Which of the following is the most accurate statement?

[A) An epidural abscess has developed]
B) The epidural block has worn off
C) The original injection was subdural
D) A radiculopathy has developed at a different level
E) She is having a reaction to the anesthetic

Item 2 of 2

36. Which of the following is the most appropriate next step?
A) Administer an analgesic and diphenhydramine
B) Administer an analgesic, intravenously, and arrange myelography
C) Contact the anesthesiologist about repeating the epidural block
D) Reassure the patient and have her make an appointment with the orthopedist for tomorrow
[E) Request an emergency MRI of the spine]
Reply
#9
Block 5
1. A 76-year-old retired plumber comes to the office with his wife because he has had progressive memory loss during the past year. His wife tells you that he has recently gotten lost in his home at night. He has also had urinary incontinence, about which he says, "It must be because of my big prostate." The patient's wife is concerned that he walks differently and often staggers, but he has not fallen. He now uses a cane. He tells you that he has recently used his wife's lorazepam for insomnia. One of his four siblings has significant memory loss. Vital signs are normal. Physical examination shows good orientation. During the mental status examination, he recalls only one of three items after 3 minutes, he is unable to do serial threes or sevens, and there is evidence of impaired judgment. He has mild ataxia with a tendency to fall to the left if he does not have support. Which of the following is the most likely presumptive diagnosis?

A) Dementia, Alzheimer type
[B) Normal-pressure hydrocephalus]
C) Parkinson dementia
D) Pseudodementia due to depression
E) Reversible drug-induced dementia

2. A 52-year-old woman comes to the office because of difficulty falling asleep. She says that she retires to her bedroom at 7:00 PM and watches television while lying on her bed. She turns out the light and the TV at 11:00 PM but lies awake until at least 2:00 AM. She then sleeps soundly until 7:00 AM when she gets up to go to work. She is divorced and lives alone. She takes no medications. Her physical examination is normal. Which of the following is the most appropriate management?

[A) Advise her to avoid watching TV in bed]
B) Advise her to drink a warm beverage before going to bed
C) Advise her to exercise lightly at 9:00 PM
D) Prescribe flurazepam
E) Prescribe temazepam

3. A 7-year-old girl is brought to the office by her mother because the girl has been awakening regularly at night in considerable distress about 1 hour after falling asleep. The mother describes her as being extremely fearful and inconsolable during these episodes. Her mother says, "She cries out something about a figure in a dark cape who is chasing after her and wants to turn her into stone. A few minutes later, she is able to go back to sleep and remembers little of what has happened the next morning." She has been in excellent health and has achieved appropriate developmental milestones. She is doing well in school and has a number of friends and playmates. She is 121 cm (4 ft) tall and weighs 22 kg (48 lb). Vital signs are: temperature 37.0°C (98.6°F), pulse 60/min, respirations 18/min and blood pressure 100/70 mm Hg. Physical examination shows a normally developed and well-nourished girl. Physical and neurologic examinations are normal. Which of the following is the most likely diagnosis?

A) Cataplexy
B) Central sleep apnea
C) Major depressive disorder
[D) Nightmares]
E) Night terrors

4. An 8-year-old boy is brought to the health center by his parents because of a 2-day history of sore throat and fever. Temperature is 38.3°C (101.0°F) and pulse is 88/min; other vital signs are normal. Physical examination shows erythema of the posterior pharynx; the tonsils are enlarged and there are a few spots of whitish exudate on the left tonsil. A few small, nontender posterior cervical lymph nodes are palpable. There is no rash. Rapid streptococcal test is negative. Which of the following is the most appropriate next step?

A) Administer intramuscular penicillin
[B) Obtain a throat culture]
C) Obtain acute-phase serum for antistreptolysin-O titer
D) Prescribe azithromycin
E) Reassure the patient's parents that he has a viral illness

5. A 16-year-old Latino boy comes to the health center because of ear pain for the past 2 days. He has been working at a local restaurant 30 to 40 hours per week through a school-sponsored vocational education program. His father left his family when the patient was a young child, and his mother died 1 year ago of breast cancer. He was declared an emancipated minor by the court after his mother's death, and he now rents a room in a home in his neighborhood. He has received care at the health center in the past for episodic illnesses and vaccinations before his mother died. Which of the following is the most accurate statement regarding obtaining consent for treatment today?

A) Can be seen because he is likely to have an infectious disease and signed consent is not necessary
B) Can be seen because his deceased mother gave signed consent for services in the past
[C) Can be seen if he signs a consent for services]
D) Cannot be seen because he has a living parent whose signed consent is required
E) Cannot be seen without consent of the court that awarded emancipated minor status

6. A 67-year-old man comes to the office because of a 1-week history of increasing cough productive of small amounts of clear sputum and shortness of breath on exertion. He has smoked one and one-half packs of cigarettes per day for the past 40 years. He quit smoking 14 months ago when he was told that he had severe lung disease. He appears to be in no acute distress. He cannot speak in full sentences without taking a breath, and he purses his lips when exhaling. The patient is 170 cm (5 ft 7 in) tall and weighs 57 kg (125 lb); BMI is 20 kg/m2. The physical examination is most likely to show which of the following?

[A) Accessory muscle use]

B) An audible, right-sided S3
C) Cyanosis of the extremities
D) Lower extremity edema
E) Wide splitting of S2

7. A 32-year-old African-American woman with a history of major depressive disorder comes to the health center because of palpitations and dizziness for the past week. She has been taking fluoxetine for the past 3 years and had been doing well until her mother died several months ago. At that time she became despondent with frequent crying and inability to sleep at night. Six weeks ago her psychiatrist increased the dose of fluoxetine but she did not think it helped. A friend recommended St. John's wort, which she began taking several weeks ago. She also takes calcium carbonate daily, occasional antihistamines for seasonal allergies, and amoxicillin-clavulanate, which was prescribed 10 days ago by another physician for a sinus infection. Vital signs now are: temperature 37.2°C (99.0°F), pulse 90/min, respirations 18/min and blood pressure 140/90 mm Hg. On physical examination she is somewhat tremulous. Thyroid is normal and chest is clear. Cardiovascular examination discloses a regular rhythm with slight tachycardia. Abdominal and neurologic examinations are normal. Which of the following is the most likely cause of her symptoms?

A) Anxiety reaction due to her mother's death
B) Hyperthyroidism
C) Interaction between fluoxetine and amoxicillin-clavulanate
D) Interaction between fluoxetine and antihistamines
[E) Interaction between fluoxetine and St. John's wort]

8. A 54-year-old African-American dispatcher comes to the office because of hip and leg pain. You have treated the patient for diabetes mellitus, emphysema and obesity. His diabetes is controlled with diet and insulin therapy and is managed by his wife who is a registered nurse. He has a 30-year history of smoking two packs of cigarettes per day and he does not want to stop. For the past 4 months he has been taking aspirin with each meal to relieve his hip and leg pain. His wife drops him off at work in the morning; he walks to the hospital to ride home with his wife. He states he has no pain at rest but walking for a few blocks causes his whole left leg to ache. Which of the following is the most likely cause of his symptoms?

A) Osteoarthritis
B) Peripheral neuropathy
[C) Peripheral vascular insufficiency]
D) Sciatic nerve radiculopathy
E) Spinal stenosis

9. A 74-year-old woman comes to the office because of constipation and blood-streaked stools for the past 3 days. She has had a 4.5-kg (10-lb) weight loss and anorexia for the past few months. She has a temperature of 37.0°C (98.6°F). Abdominal examination is normal. Rectal examination is normal except for the presence of occult blood on examination of the stool. Leukocyte count is 9000/mm3. Which of the following is the most likely diagnosis?

A) Carcinoma of the cecum
[B) Carcinoma of the sigmoid colon]
C) Ischemic colitis
D) Pseudomembranous enterocolitis
E) Ulcerative colitis

10. A 46-year-old white woman returns to the office for follow-up of abdominal pain. Two weeks ago, she came to the office because of constipation and passing two to three loose stools with mucus per day. Physical examination at that time was normal. Her weight has remained the same and temperature has been normal. She has never had an abdominal operation. Lower gastrointestinal barium study and flexible sigmoidoscopy are normal. Complete blood count is normal. Today, you review the results with the patient. Which of the following is the most appropriate recommendation to the patient?

A) Antianxiety medication
B) Antispasmodic medication
C) Consultation with a gastroenterologist
D) Consultation with a psychiatrist
[E) Fiber supplementation]

11. A 10-year-old boy is brought to the office by his mother for a periodic health evaluation. He has been your patient for several years. Recently his teachers and his school counselor have recommended that he be treated with methylphenidate because of his disruptive behavior in the classroom. He has no past history of behavioral problems. Which of the following is the most appropriate advice to the mother?

A) "Children with this problem may not have a very high IQ."
B) "Children with this problem usually grow out of it by puberty."
C) "I don't think he has attention-deficit disorder because he has not had problems in the past."
[D) "Let's get more information from the school."]
E) "Methylphenidate treatment will probably improve his grades."

12. A 24-year-old woman comes to the office for an initial prenatal visit. A home pregnancy test was positive 2 weeks ago. Her last menstrual period was 10 weeks ago. She has been healthy and has no significant medical history. She takes no medications. During the interview she becomes tearful and says, "My husband hit me several times 6 weeks ago and I'm afraid that it may happen again. He's become very loving now since he found out about the baby, but I'm still worried." Vital signs now are: temperature 36.9°C (98.4°F), pulse 80/min, respirations 20/min and blood pressure 110/70 mm Hg. Pelvic examination shows a 10-week size uterus and is otherwise normal. Ultrasonography confirms a 10-week gestation. Which of the following strategies is most appropriate?

[A) Provide her with the name and phone number of a shelter for battered women]
B) Reassure the patient that abusive behavior is less likely now that she is pregnant
C) Recommend marital counseling
D) Schedule an appointment with the husband to discuss his abusive behavior
E) Tell her that you will report this to the police if it happens again

13. A 16-year-old boy and his mother come to the office because the mother is concerned that her son may have a drug problem. You have treated him in the past for mild intermittent asthma, and he currently uses an albuterol inhaler prior to exercise. You last saw him 1 year ago for a sports physical examination. At that time, he admitted to drinking alcohol on the weekends. He also admitted to marijuana use but had planned to quit using both before track season started. The mother says that her son now seems disinterested in school and other activities in which he was previously engaged. He has quit the track team, has become irritable, and he has a new set of friends whom his mother dislikes and describes as "druggies." She acknowledges that the boy's father, from whom she is divorced, has an alcohol abuse problem. The boy's older brother abused cocaine for 2 years but is now in a drug rehabilitation program. Prior to interviewing and examining the patient, the mother pulls you aside and asks you to test her son for drugs without informing him. Which of the following is the most appropriate response?

A) Agree to do toxicologic screening but review the test results with the patient before giving the results to the mother
[B) Agree to do toxicologic screening only if, after discussing it with the patient, he grees to the test]
C) Agree to do toxicologic screening without the patient's knowledge per his mother's request
D) Decline to do toxicologic screening stating that you do not want to get caught between her and her son
E) Decline to do toxicologic screening unless under a court order

The following vignette applies to the next 2 items.


A 75-year-old woman whom you treat for obstructive lung disease comes to the office because of thoracic back pain, which has been troubling her for several weeks. There is no specific history of trauma. Her current medications include ipratropium and inhaled albuterol. She no longer smokes cigarettes and she does not use alcohol. She underwent a mastectomy 10 years ago for breast cancer. Today, vital signs are normal. Her weight is unchanged from 1 year ago. On physical examination breath sounds are diminished in intensity but normal in quality. There is no tenderness over the spinous processes. Laboratory studies in the office show a normal complete blood count and erythrocyte sedimentation rate. A compression fracture of the thoracic spine is seen on a lateral chest x-ray film. After further discussion, the patient agrees to a trial of alendronate to prevent further fractures.

Item 1 of 2

14. Regarding alendronate therapy, the patient should be told which of the following?

A) The medication should be chewed, rather than swallowed
B) The medication should be taken at bedtime
[C) The medication should be taken with a full glass of water]
D) The medication should be taken with meals
E) She should lie down immediately after taking the medication

Item 2 of 2

15. Four days later, the patient returns to the office complaining of chest pain, odynophagia and dysphagia. You decide to discontinue the alendronate. In addition, which of the following is the most appropriate diagnostic study?

A) Electrocardiography
[B) Esophagoscopy]
C) Laryngoscopy
D) MRI of the thoracic spine
E) X-ray film of the lateral neck

16. A 69-year-old woman comes to the office because of left knee pain. She says, "For the past several years my left knee has been bothering me a bit but I have been getting by. However, for the past few days it has been killing me. I was at the outlet mall 2 days ago and I was fine, but yesterday I woke up with such knee pain that I could hardly walk." She denies injuring the knee. No other joints bother her this much although she notes that her right knee occasionally hurts. The pain does not improve with ibuprofen. She has hypertension controlled with hydrochlorothiazide and a β-blocking medication, and she has mild chronic renal failure. Serum creatinine concentration 3 months ago was 2.1 mg/dL. She is 168 cm (5 ft 6 in) tall and weighs 106 kg (233 lb). Physical examination today discloses moderate effusion of the left knee. The left knee is warm to touch compared with the right knee. Range of motion is normal but there is crepitus. There is no rash or erythema of the skin. X-ray films of both knees are shown. Arthrocentesis shows:


WBC 850/mL

Gram stain Negative

Microscopic No crystals seen under polarized light


Which of the following is the most likely diagnosis?

A) Gout

[B) Osteoarthritis]
C) Osteoporosis
D) Pseudogout
E) Septic arthritis

17. A 46-year-old man with diabetes mellitus well controlled by diet alone returns to the office for a follow-up visit for migraines. You have been seeing him for the past year for the migraines, and about 2 months ago you prescribed a calcium-channel-blocking medication. His previous medications included ergotamine and propranolol, but they were discontinued because of lack of full effect. At today's visit he complains of ankle swelling. Physical examination is normal except for 2+ pitting edema. Which of the following is the most appropriate step at this time?

A) Determine serum albumin concentration
[B) Discontinue his calcium-channel-blocking medication]
C) Order determination of protein excretion in a 24-hour urine sample
D) Order echocardiography
E) Prescribe a diuretic agent

18. A 27-year-old woman comes to the clinic because of chronic dermatitis involving both ear lobes. She has pierced ears and the rash is most marked around the insertion of her earrings. She should be advised to do which of the following?

A) Avoid skin sensitizing soaps while wearing earrings
[B) Discontinue the use of metallic earring posts]
C) Have her ears repierced
D) Soak her earrings in 70% alcohol for 24 hours before using them
E) Wear clip-on earrings only

19. A 46-year-old woman whom you are treating for hypertension and hypothyroidism returns to the office for a follow-up visit. She denies dyspnea, orthopnea, nocturia or exertional chest discomfort. She states, "Doctor, I've recently found that my feet are swollen. It's becoming hard to get my shoes on. My feet don't really bother me, except that they look funny and make me feel self-conscious." The patient has no other medical problems. She has no allergies. She does not smoke cigarettes and she drinks a glass of wine every evening. Her medications are levothyroxine, extended-release nifedipine and enalapril. Blood pressure is 128/78 mm Hg. On physical examination there are no abnormalities of the skin. Neck veins are not distended. Pulmonary and cardiac examinations are normal. Liver and spleen are not palpable. She has 2+ pitting edema of both legs. The remainder of the examination is normal. Which of the following is the most likely explanation for the edema?

A) The patient has deep venous thrombosis
B) The patient has misreported her use of alcohol
C) The patient has not been taking levothyroxine
D) Use of enalapril
[E) Use of nifedipine]

20. A 45-year-old man comes to the office because he has noted a lesion on his chest. He has been generally healthy and you last saw him 5 years ago for an insurance physical examination that was normal. Medical chart notes from that visit report that his skin examination was "normal." You do not recall a lesion being present in the area that he indicates. He is unaware of when the lesion first occurred, but it was noted by a friend at a health club who saw a television report about skin cancer. Physical examination shows a well-appearing man with brown eyes and fair complexion. Vital signs are normal. Skin examination is normal except for the 1-cm lesion on his lateral abdominal wall as shown. The factor that is the best predictor of this patient's prognosis is which of the following?

A) Degree of variation in the color of the lesion
B) Diameter of the lesion
C) His complexion
D) His eye color
[E) Measurement of lesion depth]

21. A 34-year-old primigravid woman at 24 weeks' gestation returns to the office to discuss results of a fasting serum glucose study and hemoglobin A1c obtained during a visit 2 weeks ago. Medical history is significant for infertility, oligomenorrhea, and hirsutism. She conceived 3 months after starting treatment with metformin; she discontinued metformin at 10 weeks' gestation. She currently takes only a prenatal multivitamin supplement. Her mother, a maternal aunt, and a paternal aunt have type 2 diabetes mellitus. The patient is 168 cm (5 ft 6 in) tall and weighs 118 kg (260 lb); BMI is 42 kg/m2. Vital signs are: temperature 37.0°C (98.6°F), pulse 82/min, respirations 15/min, and blood pressure 112/64 mm Hg. Fasting serum glucose concentration is 120 mg/dL and hemoglobin A1c is 7.5%. Which of the following is the most appropriate management?

A) 1200-Calorie American Diabetes Association diet
B) Glyburide therapy
[C) Insulin therapy]
D) Reinitiating metformin therapy
E) Rosiglitazone therapy

22. A 28-year-old African-American woman comes to the office because of low back pain and decreased urine output. She is a semiprofessional basketball player and she is married. She had one pregnancy 12 years ago that was uncomplicated and resulted in vaginal delivery of a term female neonate. Medications include oral contraceptive pills and a corticosteroid inhaler for asthma. She is 182 cm (6 ft) tall and weighs 88 kg (195 lb). Physical examination is normal except for a palpable lower abdominal mass that extends to the umbilicus. Speculum examination discloses a 3-cm ulcerative lesion circumferentially around the external cervical os. Bimanual examination shows a firm, nodular, central pelvic mass filling the pelvis and extending cephalad to the umbilicus. Rectovaginal examination confirms these findings. Stool is negative for occult blood. Which of the following is the most appropriate recommendation?

A) Arrange for laparoscopy
B) Do a cervical biopsy
C) Do an endometrial biopsy
D) Increase Pap smear screening to every 3 months
[E) Obtain pelvic ultrasonography]

The following vignette applies to the next 3 items.

A 61-year-old woman comes to the office because of a 3-month history of urinary incontinence. You have been providing care for her and her husband since they moved to your area 2 years ago. She has a history of diabetes mellitus that was first diagnosed 2 years ago and has been very well managed by diet alone. She currently takes lorazepam at night for sleep and calcium and vitamin D supplements to prevent osteoporosis. She has declined hormone replacement therapy in the past because of a concern about breast cancer. She denies dysuria or problems with incontinence after sneezing or laughing, but she says, "When I try to pass urine, there usually isn't much, but I have to go again a few minutes later. And then, at other times, I just lose control and wet myself. I recently saw a television program on this and I think I have stress incontinence." Vital signs are: temperature 36.9°C (98.4°F), pulse 64/min, respirations 16/min and blood pressure 158/72 mm Hg. She weighs 93 kg (205 lb) and is 163 cm (5 ft 4 in) tall. Physical examination is normal except for moderate vaginal mucosal atrophy. Urinalysis done in the office shows no signs of infection.

Item 1 of 3

23. Which of the following factors in this patient's history or physical examination suggests a diagnosis other than stress incontinence as the cause for this patient's symptoms?

A) Absence of dysuria
B) Diabetes mellitus
C) Increased body mass index
[D) Pattern of urination]
E) Vaginal mucosal atrophy

Item 2 of 3

24. The most appropriate next step in management is to suggest which of the following?

A) Increased fluid consumption at night
B) Referral for bladder ultrasonography
C) Replacement of lorazepam with diphenhydramine at night for sleep
?[D) Scheduled voiding]
E) Use of a pessary

Item 3 of 3

25. The patient follows your suggestion. She returns to the office in 3 weeks and reports that she was recently incontinent while shopping with friends. She states, "I've never been so embarassed in my life! Frankly, I don't think I'm better off than when I saw you last time, and now I'm afraid to leave the house." Which of the following is the most appropriate response to the patient's comment?

A) "Are you ready to use estrogen therapy now?"
B) "Have you ever thought of using adult disposable diapers? They are very effective."
[C) "I understand your concern. Let's discuss this further and develop a plan."]
D) "I'm sorry to hear this. I'll refer you to a urologist right away."
E) "These things happen. I'm sure you can understand you're growing older."


26. A 27-year-old woman comes to the office because she recently noted a copious vaginal discharge requiring showering or bathing two or three times daily. She states that despite bathing frequently, she never feels clean. She has had no other medical problems. Results of her last Pap smear 1 year ago were normal. She is not sexually active at this time and is taking no medications. She has never been pregnant. She works as a respiratory therapist at a local community hospital and recently broke up with a boyfriend of several months. Physical examination shows no abnormalities. Pelvic examination demonstrates no vaginal discharge, bleeding, or mucosal lesions. Uterus is normal-sized and nontender. Adnexa are palpable and there are no masses. In addition to obtaining cultures for gonorrhea and chlamydia, which of the following is the most appropriate next step?

A) Determine serum estrogen concentration
B) Order antibody studies for syphilis and HIV
C) Prescribe an oral anti-trichomonal medication and antifungal cream
[D) Question the patient regarding the circumstances surrounding her recent break-up]
E) Tell the patient you will wait for the Pap smear results before prescribing anything for the discharge

27. A 77-year-old woman with breast cancer comes to the office because of a 2-week history of severe burning pain and weakness of her left arm. Two months ago, a bone scan obtained because of diffuse bony pain showed widespread metastases. The patient declined chemotherapy and asked for palliative care only. She received localized radiation therapy to left femur and right humerus for the most painful lesions. Her pain was well controlled with celecoxib and a long-acting morphine preparation until 2 weeks ago. Her only other medication is bisacodyl. The patient is alert and oriented. Vital signs are temperature 37.0°C (98.6°F), pulse 90/min, respirations 20/min, and blood pressure 110/70 mm Hg. She cannot extend her left arm above her head. There is weakness of wrist extension, flexion, and handgrip of the left upper extremity. Sensation to light touch and pinprick is decreased over the left arm. Stroking the left forearm with a cotton swab causes a painful sensation of electric shocks and heat. There are decreased biceps and brachioradialis reflexes on the left. Reflexes, strength, and sensation in the right upper extremity are normal. Neurologic examination of the lower extremities shows no abnormalities. Which of the following is the most appropriate next step in evaluation?

A) CT scan of the head
[B) Electromyography and nerve conduction studies of the left upper extremity]
C) Measurement of serum B12 (cobalamin) concentration
D) MRI of the cervical spine
E) Radionuclide bone scan

28. A 32-year-old woman, gravida 3, para 2, who is at 38 weeks' gestation, is brought to the office by her coworker 15 minutes after the patient appeared to have had a seizure at work. The coworker says the patient was working at her computer when she suddenly fell to the floor, began shaking, and had incontinence of urine. She was unresponsive to voice until approximately 5 minutes after the episode. The patient has received routine prenatal care throughout her pregnancy. She has a history of mild, persistent asthma treated with corticosteroid inhalers. Vital signs on arrival are temperature 36.7°C (98.0°F), pulse 100/min, respirations 22/min, and blood pressure 160/110 mm Hg. The patient is alert but disoriented to time and place. She has no recollection of the episode. Physical examination shows bruising of her left arm and a bite on the lower lip. Which of the following is the most accurate statement regarding the risk of harm to the fetus?

A) The fetus is at risk for developing intrauterine hypoxia
B) The fetus will die unless it is delivered immediately
C) The risk to the fetus depends on any coexisting respiratory condition
D) The risk to the fetus is minimal because the seizure was short-lived
[E) The risk to the fetus will not be increased if the seizure does not recur]

29. A 30-year-old African-American woman returns to the office for a second prenatal visit. She is 12 weeks pregnant and this is her first pregnancy. Results of laboratory studies that were ordered at her first visit show:

Blood
Hematocrit 28%
Hemoglobin 9.2 g/dL

Hemoglobin electrophoresis
Hemoglobin A1 64%
Hemoglobin S 32%
Hemoglobin A2 4%
Mean corpuscular hemoglobin (MCH) 26 pg/cell
Mean corpuscular hemoglobin concentration(MCHC) 32% Hb/cell
Mean corpuscular volume (MCV) 74 μm3

Which of the following is the most likely cause of her anemia?
A) α-thalassemia trait
B) β-thalassemia trait
C) Iron deficiency
D) Physiologic anemia of pregnancy
[E) Sickle cell trait]

30. A 17-year-old girl brings her 4-day-old neonate to the health center 1 day after discharge from the hospital. She says, "I don't think my baby is getting enough milk. He wants to nurse every 2 hours and my nipples are sore and cracked. I feel miserable." The neonate was born via vaginal delivery without complications, following a normal pregnancy. His birth weight was 3317 g (7 lb 5 oz). Physical examination shows a vigorous, active neonate with a strong sucking reflex. He has a wet diaper on arrival and has had two stools since this morning. There is jaundice of the face. In order to support this new mother during this early stage of breast-feeding, which of the following is the most appropriate advice to the patient?

A) Feed the neonate on only one breast at each feeding to ensure complete emptying of the breast
B) Insert as much of the areola as possible into the neonate's mouth to improve latching
C) Not feed the neonate more than every 3 hours to allow for better milk production
?[D) Offer formula after each feeding to ensure that the neonate is getting enough milk]
E) Switch to formula for 5 days to assist with healing of her nipples

31. A 19-year-old African-American college student comes to the student health center because of pain in her right knee and fever. She says the pain began about 4 days ago, and she does not recall injuring her knee. Her only medication is an oral contraceptive pill. She is generally healthy. Vital signs are: temperature 37.7°C (99.8°F), pulse 96/min, respirations 20/min and blood pressure 120/72 mm Hg. She appears uncomfortable. Physical examination is normal except for her right knee, which is red, swollen and tender with a tense effusion. Which of the following tests is most likely to support the diagnosis?

[A) Cervical cultures]
B) MRI of the knee
C) Plain x-ray film of the knee
D) Serum antinuclear antibody titer
E) Serum uric acid concentration

32. A 62-year-old woman who was discharged from the hospital several hours ago following evaluation of optic neuritis now has swelling of her ankles. Prior to admission, the patient had a 3-day history of loss of vision and poor color perception in her right eye. Visual acuity on admission was less than 20/200 in her right eye and was 20/40 in her left eye. The remainder of the physical examination was noncontributory. Blood pressure on admission was 140/80 mm Hg. The patient had been otherwise healthy and had been taking no medications. She has never worn corrective lenses. Family history is significant for thyroid disorder. During her hospital stay, the patient received intravenous methylprednisolone therapy for the past four days. Vital signs today are temperature 37.2°C (99.0°F), pulse 100/min, respirations 14/min, and blood pressure 150/95 mm Hg. Physical examination shows ankle edema but is otherwise unchanged from physical examination done on admission. Laboratory studies are obtained. Which of the following serum laboratory study results is most likely in this patient?

A) Decreased magnesium concentration
?[B) Decreased sodium concentration]
C) Increased calcium concentration
D) Increased glucose concentration
E) Increased potassium concentration

33. A 30-year-old white woman comes to the office for a periodic health evaluation. At this visit she asks you about a DNA test that she has heard about that can detect the presence of the adult polycystic kidney disease (APKD) gene in asymptomatic carriers. She has a family history of APKD. She has had no symptoms. Her blood pressure is normal, and renal ultrasound 2 months ago was normal. Serum creatinine concentration obtained 2 months ago was 0.9 mg/dL. Before obtaining blood for this test, it is important to explain to this patient that a positive test result would mean which of the following?

[A) She is certain to develop renal failure]
B) She may be eligible for disability
C) She may have difficulty obtaining life insurance in the future
D) She should avoid becoming pregnant
E) She should be monitored for development of liver disease

34. A pharmaceutical representative regularly comes to your office about once a month. Her company has introduced a new prescription nonsteroidal anti-inflammatory drug (NSAID) and she has brought samples today. You overhear your receptionist ask the representative to leave some samples for her own use. If the representative leaves the samples for the receptionist, the most significant concern regarding this situation is that it does which of the following?

A) It constitutes practicing medicine without a license

B) It diverts samples intended for use by patients

C) It may constitute a liability risk

D) It promotes the use of a more expensive drug over available alternative medications

[E) It provides the representative the opportunity to exert undue influence]

35. A 54-year-old woman comes to the office for her annual health maintenance examination. At her last visit 11 months ago she reported hot flushes but says they have now resolved. She takes no medications. She is employed as an executive for a computer manufacturing company. Her stress level has increased during the past 9 months because of her company's financial difficulties and she now smokes two packs of cigarettes daily. She is 170 cm (5 ft 7 in) tall and weighs 63 kg (140 lb), which is an increase of 4 kg (9 lb) since her last visit. She plans to begin an intensive aerobics program to stop her weight gain. The patient should be advised that initiation of this exercise program will put her most at risk for which of the following?

A) Arthritis
B) Exercise-induced asthma
C) Gastroesophageal reflux disease
[D) Stress fractures]
E) Stroke

36. A 78-year-old retired autoworker from Puerto Rico is brought to the office by his son because of a 4- to 7-kg (9- to 15-lb) weight loss in the past few months. You have been treating this patient for hypertension with hydrochlorothiazide for the past 3 years. Today he reports that he eats only one or two bites of food and feels full and that he has a low energy level. He feels he is just "too old." He has never smoked cigarettes and rarely drinks alcoholic beverages. Which of the following is the most appropriate question to elicit further history that would be helpful in the differential diagnosis of this patient?

A) "Are you having headaches?"
B) "Did you stop taking your medication?"
C) "Have you had a change in bowel habits?"
D) "Have you had any visual changes lately?"
[E) "What has been going on in your life recently?]
Reply
#10

ANSWERS FOR NBME TEST 1

1 A
2 A
3 E
4 A
5 D
6 D
7 E
8 A
9 E
10 D
11 E
12 B
13 D
14 C
15 B
16 A
17 E
18 A
19 E
20 B
21 A
22 B
23 E
24 C
25 B
26 C
27 D
28 E
29 A
30 B
31 C
32 B
33 C
34 E
35 C
36 C
....................
ANSWERS FOR TEST 2

1.c
2.d
3.a
4.e
5.e
6.e
7.e
8.b
9.e
10.d
11.b
12.a
13.d
14.c
15.c
16.b
17.b
18.a
19.a
20.d
21.a
22.e
23.c
24.b
25.e
26.e
27.e
28.c
29.a
30.a
31.d
32.c
33.d
34.b
35.c
36.e
......................

ANSWERS for test 3

1 b
2e
3c
4d
5e
6c
7c
8e
9a
10e
11a
12c
13c
14e
15b
16b
17a
18e
19e
20a
21a
22b
23b
24c
25d
26a
27c
28b
29d
30c
31e
32c
33a
34a
35b
36a
...........................
ANSWERS for test 4

1c
2b
3d
4c
5e
6e
7e
8e
9b
10e
11c
12b
13b
14e
15e
16e
17d
18b
19b
20c
21b
22a
23c
24a
25d
26d
27e
28d
29e
30a
31c
32d
33a
34b
35a
36e
...........................
ANSWERS for test 5

1 b
2 a
3 d
4 b
5 c
6 a
7 e
8 c
9 b
10 e
11 d
12 a
13 b
14 c
15 b
16 e
17 c
18 b
19 e
20 e
21 c
22 b
23 d
24 b
25 c
26 d
27 d
28 a
29 e
30 b
31 a
32 d
33 a
34 e
35 d
36 e
Reply
« Next Oldest | Next Newest »


Forum Jump: